Search results

  • The '''Power Mean Inequality''' is a generalized form of the multi-variable [[Arithmetic Mean-Geometric Mean]] I ...ath>, the power mean with exponent <math>t</math>, where <math>t\in\mathbb{R}</math>, is defined by
    3 KB (606 words) - 23:59, 1 July 2022
  • ...ts, the principle may be referred to as the '''Dirichlet box principle'''. A common phrasing of the principle uses balls and boxes and is that if <math> ...nhole principle is as follows: suppose for contradiction that there exists a way to place <math>n</math> balls into <math>k</math> boxes where <math>n>k
    11 KB (1,985 words) - 21:03, 5 August 2023
  • ...generally concerned with finding the number of combinations of size <math>r</math> from an original set of size <math>n</math> ...ons are, their various types, and how to calculate each type! It serves as a great introductory video to combinations, permutations, and counting proble
    4 KB (615 words) - 11:43, 21 May 2021
  • ...f and only if there exists a constant <math>t</math> such that <math>a_i = t b_i</math> for all <math>1 \leq i \leq n</math>, or if one list consists of ...there exists a scalar <math>t</math> such that <math>\overrightarrow{v} = t \overrightarrow{w}</math>, or if one of the vectors is zero. This formulati
    13 KB (2,048 words) - 15:28, 22 February 2024
  • ...[equality condition | equality case]] of [[Ptolemy's Inequality]]. Ptolemy's theorem frequently shows up as an intermediate step in problems involving i ...ABCD</math> with side lengths <math>{a},{b},{c},{d}</math> and [[diagonal]]s <math>{e},{f}</math>:
    7 KB (1,198 words) - 20:39, 9 March 2024
  • ...r theory include the [[Birch and Swinnerton-Dyer Conjecture]] and [[Fermat's Last Theorem]]. ...in some subfield (like the reals or the rationals). One also needs to add a limit point, called the point at infinity. As <math>x\to \infty</math>, the
    5 KB (849 words) - 16:14, 18 May 2021
  • ...integer]]s are [[divisibility | divisible]] by particular other [[integer]]s. All of these rules apply for [[Base number| base-10]] ''only'' -- other b https://youtu.be/6xNkyDgIhEE?t=1699
    8 KB (1,315 words) - 18:18, 2 March 2024
  • '''Jensen's Inequality''' is an inequality discovered by Danish mathematician Johan Jen ...[convex function]] of one real variable. Let <math>x_1,\dots,x_n\in\mathbb R</math> and let <math>a_1,\dots, a_n\ge 0</math> satisfy <math>a_1+\dots+a_n
    3 KB (623 words) - 13:10, 20 February 2024
  • == Pascal's Identity == Pascal's Identity states that
    12 KB (1,996 words) - 12:01, 18 May 2024
  • A '''circle''' is a geometric figure commonly used in Euclidean [[geometry]]. {{asy image|<asy>unitsize(2cm);draw(unitcircle,blue);</asy>|right|A basic circle.}}
    9 KB (1,581 words) - 18:59, 9 May 2024
  • ...to J</math> and <math>f:J \to \mathbb{R}</math>. Let <math>h:I \to \mathbb{R}</math> such that <math>h(x) = f(g(x)) \forall x \in I</math>. If <math>x_ ...<math>h'(x_0)</math>,<math>f'(g(x_0))</math>, and <math>g'(x_0)</math> is a matrix.)
    12 KB (2,377 words) - 11:48, 22 July 2009
  • ...>p</math> [[Majorization|majorizes]] a sequence <math>q</math>, then given a set of positive reals <math>x_1,x_2,\cdots,x_n</math>: ...ath> majorizes <math>(4,2)</math> (as <math>5>4, 5+1=4+2</math>), Muirhead's inequality states that for any positive <math>x,y</math>,
    8 KB (1,346 words) - 12:53, 8 October 2023
  • The '''Fundamental Theorem of Calculus''' establishes a link between the two central operations of [[calculus]]: [[derivative|diffe This section is for people who know what [[integral]]s are but don't know the Fundamental Theorem of Calculus yet, and would like to try to figu
    11 KB (2,082 words) - 15:23, 2 January 2022
  • ...y, but also most abstractly, a vector is any object which is an element of a given vector space. ...(x\,\,y\,\,z\,\,...)</math>. The set of vectors over a [[field]] is called a [[vector space]].
    7 KB (1,265 words) - 13:22, 14 July 2021
  • ...would be a pain to have to calculate any time you wanted to use it (say in a comparison of large numbers). Its natural logarithm though (partly due to ...ly 7 digits before the decimal point. Comparing the logs of the numbers to a given precision can allow easier comparison than computing and comparing th
    4 KB (680 words) - 12:54, 16 October 2023
  • ...ts of unity come up when we examine the [[complex number|complex]] [[root]]s of the [[polynomial]] <math> x^n=1 </math>. ...making <math>r^n=1\Rightarrow r=1</math> (magnitude is always expressed as a positive real number). This leaves us with <math>e^{ni\theta} = e^{2\pi ik
    3 KB (558 words) - 21:36, 11 December 2011
  • ...ficient way of finding the sums of [[root]]s of a [[polynomial]] raised to a power. They can also be used to derive several [[factoring]] [[identity|id Consider a polynomial <math>P(x)</math> of degree <math>n</math>,
    4 KB (690 words) - 13:11, 20 February 2024
  • ...n [[real number]] <math>x</math> can be approximated by [[rational number]]s. Of course, since the rationals are dense on the real line, we, surely, can ...th> can be approximated by a rational number <math>\frac{p}{q}</math> with a given denominator <math>q\ge 1</math> with an error not exceeding <math>\fr
    7 KB (1,290 words) - 12:18, 30 May 2019
  • ...s and angles of triangles through the '''trigonometric functions'''. It is a fundamental branch of mathematics, and its discovery paved the way towards ...[[Law of Sines]] and the [[Law of Cosines]]; many more, such as [[Stewart's Theorem]], are most easily proven using trigonometry. In algebra, expressio
    8 KB (1,217 words) - 20:15, 7 September 2023
  • ...ric mean]], and [[harmonic mean]] of a set of [[positive]] [[real number]]s <math>x_1,\ldots,x_n</math> that says: ..._1}+\cdots+\frac{1}{x_n}} \ge \sqrt[n_4]{\frac{x_1^{n_4}+\cdots+x_a^{n_4}}{a}}</cmath> where <math>n_1>1,~~0<n_2<1,~~-1<n_3<0,~~n_4<-1</math>, and <math
    5 KB (912 words) - 20:06, 14 March 2023
  • ...every [[even integer]] greater than two is the sum of two [[prime number]]s. The conjecture has been tested up to 400,000,000,000,000. Goldbach's conjecture is one of the oldest unsolved problems in [[number theory]] and
    7 KB (1,201 words) - 16:59, 19 February 2024
  • ...e lengths of the [[line segment]]s formed when two [[line]]s [[intersect]] a [[circle]] and each other. ...of the lines is [[tangent line|tangent]] to the circle while the other is a [[secant line|secant]] (middle figure). In this case, we have <math> AB^2 =
    5 KB (827 words) - 17:30, 21 February 2024
  • We say a [[nonincreasing]] [[sequence]] of [[real number]]s <math> a_1, \ldots ,a_n</math> '''majorizes''' another nonincreasing sequen ...also say that this is the case if and only if for all <math> t \in \mathbb{R} </math>,
    2 KB (288 words) - 22:48, 5 July 2023
  • ...an actual [[AMC]] (American Mathematics Competitions 8, 10, or 12) exam. A number of '''Mock AMC''' competitions have been hosted on the [[Art of Prob == Tips for Writing a Mock AMC ==
    51 KB (6,175 words) - 20:58, 6 December 2023
  • ...is article is to explain the basics of modular arithmetic while presenting a progression of more difficult and more interesting problems that are easily https://youtu.be/7an5wU9Q5hk?t=777
    15 KB (2,396 words) - 20:24, 21 February 2024
  • ...or <math>a \equiv b</math> (mod <math>n</math>), if the difference <math>{a - b}</math> is divisible by <math>n</math>. ...Z}_n</math> for short). This structure gives us a useful tool for solving a wide range of number-theoretic problems, including finding solutions to [[D
    14 KB (2,317 words) - 19:01, 29 October 2021
  • In quadrilateral <math> ABCD , \angle B </math> is a right angle, diagonal <math> \overline{AC} </math> is perpendicular to <mat ...f <math> \mathcal{A}. </math> Find the number of possible values of <math> S. </math>
    7 KB (1,173 words) - 03:31, 4 January 2023
  • ...st integer <math> n </math> less than 1000 such that <math> S_n </math> is a [[perfect square]]. ...ldots,</math> and <math>2^{n-1}-2^{n-2} = 2^{n-2}</math> elements of <math>S</math> that are divisible by <math>2^1</math> but not by <math>2^2</math>.
    10 KB (1,702 words) - 00:45, 16 November 2023
  • This article provides a short list of commonly used LaTeX symbols. .../math> on the web, (technically an AJAX library simulating it.)) maintains a [http://docs.mathjax.org/en/latest/tex.html#supported-latex-commands list o
    16 KB (2,324 words) - 16:50, 19 February 2024
  • ...area of rhombus <math> \mathcal{T}</math>. Given that <math> K </math> is a [[positive integer]], find the number of possible values for <math> K</math pair A=(0,0), B=(4.2,0), C=(5.85,-1.6), D=(4.2,-3.2), EE=(0,-3.2), F=(-1.65,-1.6),
    5 KB (730 words) - 15:05, 15 January 2024
  • {{AMC12 Problems|year=2003|ab=A}} <math> \mathrm{(A) \ } 0\qquad \mathrm{(B) \ } 1\qquad \mathrm{(C) \ } 2\qquad \mathrm{(D) \
    13 KB (1,955 words) - 21:06, 19 August 2023
  • ...ms, see [[Zermelo-Fraenkel Axioms]]. In this article we shall present just a brief discussion of the most common properties of sets and operations relat ...g: <math>\{1,4,5,3,24,4,4,5,6,2\}</math> Such an entity is actually called a multiset.
    11 KB (2,021 words) - 00:00, 17 July 2011
  • '''Newman's Tauberian Theorem''' is a [[tauberian theorem]] Let <math>f:(0,+\infty)\to\mathbb C</math> be a bounded function. Assume that
    6 KB (1,034 words) - 07:55, 12 August 2019
  • ...members left over. The director realizes that if he arranges the group in a formation with 7 more rows than columns, there are no members left over. Fi ...the number of students is <math>n(n + 7)</math> which must be 5 more than a perfect square, so <math>n \leq 14</math>. In fact, when <math>n = 14</mat
    8 KB (1,248 words) - 11:43, 16 August 2022
  • ...ath>1 + ir</math> and <math>1 - ir</math>. Their product is <math>P = 1 + r^2 = 1 + \sqrt{2006}</math>. <math>44^2 = 1936 < 2006 < 2025 = 45^2</math> If you think of each part of the product as a quadratic, then <math>((x-1)^2+\sqrt{2006})</math> is bound to hold the two
    4 KB (686 words) - 01:55, 5 December 2022
  • pair A = origin; pair C = rotate(15,A)*(A+dir(-50));
    13 KB (2,129 words) - 18:56, 1 January 2024
  • ...eated <math>8</math> more times. After the last fold, the strip has become a stack of <math>1024</math> unit squares. How many of these squares lie belo ...number of squares below the <math>n</math> square after the final fold in a strip of length <math>2^{k}</math>.
    6 KB (899 words) - 20:58, 12 May 2022
  • ...th> 7+77+777+7+7=875 </math> could be obtained from eight <math> 7 </math>'s in this way. For how many values of <math> n </math> is it possible to inse ...</math>. Then the question is asking for the number of values of <math>n = a + 2b + 3c</math>.
    11 KB (1,857 words) - 21:55, 19 June 2023
  • ...neither the [[perfect square | square]] nor the [[perfect cube | cube]] of a positive integer. Find the 500th term of this sequence. ...9}{58}</math> as large as each interior angle of <math>P_2^{}</math>. What's the largest possible value of <math>s_{}^{}</math>?
    6 KB (870 words) - 10:14, 19 June 2021
  • ...n for office, a candidate made a tour of a country which we assume lies in a plane. On the first day of the tour he went east, on the second day he went The table below displays some of the results of last summer's Frostbite Falls Fishing Festival, showing how many contestants caught <math
    8 KB (1,275 words) - 06:55, 2 September 2021
  • ...math> rectangles, of which <math>s</math> are squares. The number <math>s/r</math> can be written in the form <math>m/n,</math> where <math>m</math> an ...he two-digit number to the left of the three-digit number, thereby forming a five-digit number. This number is exactly nine times the product Sarah sho
    7 KB (1,098 words) - 17:08, 25 June 2020
  • ...than the mean of <math>\mathcal{S}</math>. Find the mean of <math>\mathcal{S}</math>. ...and <math>c</math> is not divisible by the square of any prime. Find <math>a+b+c</math>.
    7 KB (1,212 words) - 22:16, 17 December 2023
  • ...plate will contain at least one palindrome (a three-letter arrangement or a three-digit arrangement that reads the same left-to-right as it does right- ...gram shows twenty congruent circles arranged in three rows and enclosed in a rectangle. The circles are tangent to one another and to the sides of the r
    8 KB (1,374 words) - 21:09, 27 July 2023
  • ...rom left to right, each pair of consecutive digits of <math>N</math> forms a perfect square. What are the leftmost three digits of <math>N</math>? Each of the 2001 students at a high school studies either Spanish or French, and some study both. The numb
    8 KB (1,282 words) - 21:12, 19 February 2019
  • ...<math>P</math>, one of the points of intersection, a line is drawn in such a way that the chords <math>QP</math> and <math>PR</math> have equal length. ...l("$P$",P,1.5*dir(80)); label("$R$",R,NE); label("12",waypoint(O1--O2,0.4),S);</asy>
    13 KB (2,149 words) - 18:44, 5 February 2024
  • ...r arc <math>AB</math> is a rational number. If this number is expressed as a fraction <math>\frac{m}{n}</math> in lowest terms, what is the product <mat pair A=(-0.91,-0.41);
    19 KB (3,221 words) - 01:05, 7 February 2023
  • ...math> is a root of <math>z^6+z^3+1</math>, then <math>0=(r^3-1)(r^6+r^3+1)=r^9-1</math>. The polynomial <math>x^9-1</math> has all of its roots with [[a ...\circ </math>, then <math>r^6+r^3+1=3</math>. (When we multiplied by <math>r^3 - 1</math> at the beginning, we introduced some extraneous solutions, and
    3 KB (430 words) - 19:05, 7 February 2023
  • ...math> is <math>12 \mbox { cm}^2</math>. These two faces meet each other at a <math>30^\circ</math> angle. Find the [[volume]] of the tetrahedron in <mat path3 rightanglemark(triple A, triple B, triple C, real s=8)
    6 KB (947 words) - 20:44, 26 November 2021
  • ...tail is immediately followed by a head, a head is immediately followed by a head, and etc. We denote these by <tt>TH</tt>, <tt>HH</tt>, and etc. For ex ...tches to <tt>H</tt> four times; hence it follows that our string will have a structure of <tt>THTHTHTH</tt>.
    4 KB (772 words) - 21:09, 7 May 2024
  • ...XY = WZ</math>, <math>PQ = PQ</math> and the [[area]]s of the [[trapezoid]]s <math>PQZW</math> and <math>PQYX</math> are the same, then the heights of t ...is equal to <math>[PXYQ]=\frac{19}{2}* \frac{XY+87}{2}=\frac{19}{2}* \frac{a+b+106}{2}</math>. Setting these equal to each other and solving gives <math
    3 KB (530 words) - 07:46, 1 June 2018
  • ...ath> and <math>b</math> are integers such that <math>x^2 - x - 1</math> is a factor of <math>ax^{17} + bx^{16} + 1</math>. 0 = ax^{17} + bx^{16} + 1 = a(F_{17}\cdot x + F_{16}) + b(F_{16}\cdot x + F_{15}) + 1 &\Longrightarrow (a
    10 KB (1,585 words) - 03:58, 1 May 2023
  • ...lex number]] of the form <math>r+si</math>, where <math>r</math> and <math>s</math> are integers, can be uniquely expressed in the base <math>-n+i</math <center><math>r+si=a_m(-n+i)^m+a_{m-1}(-n+i)^{m-1}+\cdots +a_1(-n+i)+a_0</math></center>
    2 KB (408 words) - 17:28, 16 September 2023
  • ...ach between 1 and 1000 inclusive, with repetitions allowed. The sample has a unique [[mode]] (most frequent value). Let <math>D</math> be the difference ...be <math>M</math>, and the sum of the numbers <math>\neq x</math> be <math>S</math>. Then
    5 KB (851 words) - 18:01, 28 December 2022
  • .../math> can be written in the form <math>ax+2y+c=0_{}^{}</math>. Find <math>a+c_{}^{}</math>. pair P=(-8,5),Q=(-15,-19),R=(1,-7),S=(7,-15),T=(-4,-17);
    8 KB (1,319 words) - 11:34, 22 November 2023
  • ...^{}_{}</math> are not necessarily distinct. To write the elements of <math>S^{}_{}</math> as fractions in lowest terms, how many different numerators ar ...ere aren't any numbers which are multiples of <math>37^2</math>, so we can't get numerators which are multiples of <math>37</math>. Therefore <math>648
    2 KB (277 words) - 20:45, 4 March 2024
  • ...enny and Kenny can see each other again. If <math>t\,</math> is written as a fraction in lowest terms, what is the sum of the numerator and denominator? ...100).</math> Then at time <math>t</math>, they end up at points <math>(-50+t,100)</math> and <math>(-50+3t,-100).</math>
    8 KB (1,231 words) - 20:06, 26 November 2023
  • For certain real values of <math>a, b, c,</math> and <math>d_{},</math> the equation <math>x^4+ax^3+bx^2+cx+d Since the [[coefficient]]s of the [[polynomial]] are real, it follows that the non-real roots must com
    3 KB (451 words) - 15:02, 6 September 2021
  • ...math>f</math> defined by <math>f(x)= \frac{ax+b}{cx+d}</math>, where <math>a</math>,<math>b</math>,<math>c</math> and <math>d</math> are nonzero real nu ...in the domain. Substituting the function definition, we have <math>\frac {a\frac {ax + b}{cx + d} + b}{c\frac {ax + b}{cx + d} + d} = x</math>, which r
    11 KB (2,063 words) - 22:59, 21 October 2023
  • A stack of <math>2000</math> cards is labelled with the integers from <math>1 ...r logic as Solution 1, we find that 1999 has position <math>1024</math> in a <math>2048</math> card stack, where the fake cards towards the front.
    15 KB (2,673 words) - 19:16, 6 January 2024
  • ...r</math> times as large as angle <math>APQ,</math> where <math>r</math> is a positive real number. Find <math>\lfloor 1000r \rfloor</math>. pair A,B,C,P,Q;
    8 KB (1,275 words) - 03:04, 27 February 2022
  • ...), I=incenter(A,B,C), D=IP((0,I.y)--(20,I.y),A--B), E=IP((0,I.y)--(20,I.y),A--C); D(MP("A",A,N)--MP("B",B)--MP("C",C)--cycle); D(MP("I",I,NE)); D(MP("E",E,NE)--MP("D",D
    9 KB (1,540 words) - 08:31, 1 December 2022
  • ...math>2</math> representation has more <math>1</math>'s than <math>0</math>'s. Find the [[remainder]] when <math> N </math> is divided by <math>1000</mat ...</math> notation, with <math>k+1</math> of the digits being <math>1</math>'s.
    4 KB (651 words) - 19:42, 7 October 2023
  • ...<math>c</math> are positive integers. Find <math>\left(p+q+r+s\right)\left(a+b+c\right)</math>. ...y=\frac{2x}{3}</math>. We can find the number of such paths using a Pascal's Triangle type method below, computing the number of paths to each point tha
    7 KB (1,127 words) - 13:34, 19 June 2022
  • ...ine{ST}</math> with <math>S</math> on <math>\overline{PR}</math> and <math>T</math> on <math>\overline{QR}</math>, such that <math>\overline{ST}</math> pair P = (0,0), Q = (90, 0), R = (0, 120), S=(0, 60), T=(45, 60), U = (60,0), V=(60, 40), O1 = (30,30), O2 = (15, 75), O3 = (70, 10
    7 KB (1,112 words) - 02:15, 26 December 2022
  • ...<math>r</math> are relatively prime, and <math>r>0</math>. Find <math>m+n+r</math>. Thus <math>r=\frac{-1+\sqrt{161}}{40}</math>, and so the final answer is <math>-1+161+40
    6 KB (1,060 words) - 17:36, 26 April 2024
  • ...ath> be a prime number and let <math>s</math> be an integer with <math>0 < s < p </math>. Prove that there exist integers <math>m</math> and <math>n</ma <math> \left\{ \frac{sm}{p} \right\} < \left\{ \frac{sn}{p} \right\} < \frac{s}{p} </math>
    3 KB (506 words) - 17:54, 22 June 2023
  • Let <math>f(x)</math> be a non-constant polynomial in <math>x</math> of degree <math>d</math> with Let <math>g(n) = f(n^2)</math>, then <math>g(n)</math> is a polynomial of degree <math>2</math> or
    9 KB (1,699 words) - 13:48, 11 April 2020
  • ...th>, <math>SBF</math>, <math>TCF</math>, and <math>TDE</math> pass through a common point. ...imilarity, and <math>X</math> is the center of spiral similarity for <math>A,E,B,</math> and <math>F</math>.
    5 KB (986 words) - 22:46, 18 May 2015
  • A '''Dedekind domain''' is a [[integral domain]] <math>R</math> satisfying the following properties: * <math>R</math> is a [[noetherian]] [[ring]].
    9 KB (1,648 words) - 16:36, 14 October 2017
  • ...h>m</math> and <math>n</math> are [[relatively prime]] [[positive integer]]s, compute the sum of the positive [[prime number | prime]] [[divisor | facto ..._{10}+a_{11}+a_{12}+...</math>. If the sum of all distinct values of <math>S</math> is <math>\frac{m}{n}</math> where <math>m</math> and <math>n</math>
    5 KB (744 words) - 19:46, 20 October 2020
  • ...se variables - but there is no obvious way of computing them. We will show a different solution. ...\alpha\beta\gamma=r</math>. Then we can write <math>P(x)=x^3 - px^2 - qx - r</math>.
    3 KB (568 words) - 15:50, 3 April 2012
  • ...</math> with a variable radius that is tangent to <math>AB</math> at <math>T</math>. Let <math>P</math> be the point such that circle <math>O</math> is ...ee below an attempted solution to understand why this problem doesn't have a solution:
    3 KB (541 words) - 17:32, 22 November 2023
  • ...^{7}</math>. If <math>m</math> is the number of elements in <math>\mathcal{S}</math>, compute <math>\star(m)</math>. ...\sqrt{7}</math>, <math>CA=1</math>, and <math>AB=3</math>. If <math>\angle A=\frac{\pi}{n}</math> where <math>n</math> is an integer, find the remainder
    8 KB (1,355 words) - 14:54, 21 August 2020
  • .../math> such that the two polynomials <math>x^2+ax+1</math> and <math>x^2+x+a</math> have at least one root in common. Let this root be <math>r</math>. Then we have
    2 KB (346 words) - 23:20, 18 January 2023
  • A twin prime pair is a set of two primes <math>(p, q)</math> such that <math>q</math> is <math>2</ <math>\mathrm{(A)}\, 4</math>
    30 KB (4,794 words) - 23:00, 8 May 2024
  • ..., then we can conclude that <math>m</math> is a prime. Since there must be a factor of <math>m</math> less than <math>\sqrt{m}</math>. ...t let <math>r=\lfloor\sqrt{n/3}\rfloor</math>, then we can write <math>n=3(r+\epsilon)^2(0\leq\epsilon< 1)</math>, so <math>h=6r\epsilon+3\epsilon^2\leq
    2 KB (430 words) - 13:03, 24 February 2024
  • ...fact of Euclidean [[geometry]]. Certain fixed orders and distance [[ratio]]s hold among these points. In particular, <math>\overline{OGNH}</math> and < ...s examine what else this transformation, which we denote as <math>\mathcal{S}</math>, will do.
    59 KB (10,203 words) - 04:47, 30 August 2023
  • {{AMC10 Problems|year=2003|ab=A}} <math> \mathrm{(A) \ } 0\qquad \mathrm{(B) \ } 1\qquad \mathrm{(C) \ } 2\qquad \mathrm{(D) \
    13 KB (1,900 words) - 22:27, 6 January 2021
  • ...rcumscribed circle. What is the radius, in inches, of the circle? <!-- don't remove the following tag, for PoTW on the Wiki front page--></onlyinclude> <math> \mathrm{(A) \ } \frac{3\sqrt{2}}{\pi}\qquad \mathrm{(B) \ } \frac{3\sqrt{3}}{\pi}\qqu
    2 KB (414 words) - 13:48, 4 April 2024
  • A [[circle]] has center on the side <math>AB</math> of the [[cyclic quadrilat ...>O</math> be the center of the circle mentioned in the problem. Let <math>T</math> be the second intersection of the circumcircle of <math>CDO </math>
    4 KB (684 words) - 07:28, 3 October 2021
  • ...class is making a golf trophy. He has to paint <math>300</math> dimples on a golf ball. If it takes him <math>2</math> seconds to paint one dimple, how <math> \mathrm{(A) \ 4 } \qquad \mathrm{(B) \ 6 } \qquad \mathrm{(C) \ 8 } \qquad \mathrm{(D)
    13 KB (1,994 words) - 13:04, 18 February 2024
  • ...x + d </math>, where <math> a, b, c, d </math> are [[integer]]s and <math> a \neq 0 </math>. Suppose that <math> xP(x) = yP(y) </math> for infinitely m a(x^4 - y^4) + b(x^3 - y^3) + c(x^2 - y^2) + d(x - y) = 0
    4 KB (662 words) - 19:51, 8 September 2009
  • A '''point''' is associated with a cartesian coordinate pair in Asymptote. There are two useful functions tha ...ewise cubic function of a parameter <math>t</math>, parameterized as <math>t</math> ranges from <math>0</math> to the number of nodes (say <math>n</math
    7 KB (1,205 words) - 21:38, 26 March 2024
  • .../math> is the set of all elements of <math>S</math> which are not in <math>A.</math>) **<math>A</math> must have either 0 or 6 elements, probability: <math>\frac{2}{2^6} =
    8 KB (1,367 words) - 11:48, 23 October 2022
  • Let <math>ABC </math> be a triangle such that \left( \cot \frac{A}{2} \right)^2 + \left( 2 \cot \frac{B}{2} \right)^2 + \left( 3 \cot \frac{C
    2 KB (298 words) - 22:32, 6 April 2016
  • ...ber]]s. Determine all functions <math> f : \mathbb{R} \rightarrow \mathbb{R} </math> such that ...math>, then we obtain <math>f(x^2) = xf(x) </math>. Therefore the problem's condition becomes
    3 KB (490 words) - 22:09, 18 September 2018
  • ...tive integers with three distinct digits. Compute the remainder when <math>S</math> is divided by <math>1000</math>. <math>A, B, C, D,</math> and <math>E</math> are collinear in that order such that <
    6 KB (1,100 words) - 22:35, 9 January 2016
  • ...>P's</math> is a word in Zuminglish if and only if between any two <math>O's</math> there appear at least two consonants. Let <math>N</math> denote the ...te the number of <math>n</math>-letter words ending in a vowel followed by a constant (<tt>VC</tt> - the only other combination, two vowels, is impossib
    5 KB (795 words) - 16:03, 17 October 2021
  • ...oints of <math>AE</math>, <math>BC</math>, and <math>CD</math>. The plane's intersection with the pyramid has an area that can be expressed as <math>\s Note first that the intersection is a [[pentagon]].
    7 KB (1,034 words) - 21:56, 22 September 2022
  • .../math>, and <math>\omega</math> is [[externally tangent]] to <math>\omega_{A},</math> <math>\omega_{B},</math> and <math>\omega_{C}</math>. If the sides pair A,B,C,X,Y,Z,P,Q,R;
    11 KB (2,099 words) - 17:51, 4 January 2024
  • ...>b</math>, and <math>c</math> be the lengths of a triangle whose area is ''S''. Prove that <math>a^2 + b^2 + c^2 \ge 4S\sqrt{3}</math>
    5 KB (860 words) - 13:12, 13 February 2024
  • ...m Vandervelde'') Let <math>n</math> be a [[positive]] [[integer]]. Define a [[sequence]] by setting <math>a_1 = n</math> and, for each <math>k>1</math> ...integers, so <math>b_{k+1} \le b_k</math>. As the <math>b_k</math>'s form a [[non-increasing]] sequence of positive integers, they must eventually beco
    6 KB (1,204 words) - 20:06, 23 August 2023
  • (''Reid Barton'') An ''animal'' with <math>n</math> ''cells'' is a connected figure consisting of <math>n</math> equal-sized [[Square (geometr ...two or more dinosaurs. Find with proof the [[maximum]] number of cells in a primitive dinosaur.
    10 KB (1,878 words) - 14:56, 30 June 2021
  • ...ga</math>. Let <math>P_A</math> and <math>Q_A</math> denote the [[center]]s of <math>\omega_A</math> and <math>\Omega_A</math>, respectively. Define p <cmath>8P_AQ_A \cdot P_BQ_B \cdot P_CQ_C \le R^3,</cmath>
    7 KB (1,274 words) - 15:11, 31 August 2017
  • ...very triangle is [[cyclic]], every triangle has a circumscribed circle, or a [[circumcircle]]. ==Formula for a Triangle==
    4 KB (729 words) - 16:52, 19 February 2024
  • ...his allows you to use the same code in many source files by just including a single line in each source file. ...font size, which is 10pt by default but can be increased to 11pt or 12pt. A reference on other options for this command can be found [http://www.nada.k
    30 KB (5,171 words) - 10:16, 4 April 2021
  • | <math>a^{i+1}_3</math>||a^{i+1}_3||<math>x^{3^2}</math>||x^{3^2} Notice that we can apply both a subscript and a superscript at the same time. For subscripts or superscripts with more than
    12 KB (1,898 words) - 15:31, 22 February 2024
  • ...both choices require the same amount of time. What is the [[ratio]] of Yan's distance from his home to his distance from the stadium? <math>\mathrm{(A)}\ \frac 23\qquad \mathrm{(B)}\ \frac 34\qquad \mathrm{(C)}\ \frac 45\qquad
    5 KB (804 words) - 14:55, 21 August 2022
  • ...had ridden for twice the length of time as Mike and at four-fifths of Mike's rate. How many miles had Mike ridden when they met? (\mathrm {A}) \ 4 \qquad (\mathrm {B}) \ 5 \qquad (\mathrm {C})\ 6 \qquad (\mathrm {D})
    1,006 bytes (166 words) - 21:18, 3 July 2013
  • ...>b</math> be positive integers such that <math>ab + 1</math> divides <math>a^{2} + b^{2}</math>. Show that \frac {a^{2} + b^{2}}{ab + 1}
    4 KB (720 words) - 12:26, 7 April 2024
  • Isabella's house has 3 bedrooms. Each bedroom is 12 feet long, 10 feet wide, and 8 fee <math> \mathrm{(A) \ } 678\qquad \mathrm{(B) \ } 768\qquad \mathrm{(C) \ } 786\qquad \mathrm{
    12 KB (1,814 words) - 12:58, 19 February 2020
  • The bundled <code>markers</code> package provides a useful function for marking angles. The exact same function exists in the < ...e functions, you can replace <code>line l1, line l2</code> with <code>pair A, pair O, pair B</code>, which would mark the angle <math>\angle AOB</math>.
    4 KB (646 words) - 21:18, 26 March 2024
  • ...two-sided ideals of rings are similar to [[normal subgroup]]s of [[group]]s. ...xa \in \alpha</math>, for all <math>x\in R</math> and <math>a\in \mathfrak{a}</math>. Symbolically, this can be written as
    8 KB (1,389 words) - 23:44, 17 February 2020
  • <math> \textbf{(A) \ } \frac {100(M - N)}{M} \qquad \textbf{(B) \ } \frac {100(M - N)}{N} \qq A rectangular field is half as wide as it is long and is completely enclosed
    23 KB (3,641 words) - 22:23, 3 November 2023
  • ...\circ}</math>, angles <math>B</math> and <math>D</math> are [[right angle]]s, <math>AB = 13</math>, and <math>AD = 46</math>. Then <math>AC=</math> <math>\mathrm{(A)}\ 60
    3 KB (391 words) - 14:30, 5 July 2013
  • <math>\mathrm{(A)}\ 250,\!000 Let the roots be <math>r,s,r + s</math>, and let <math>t = rs</math>. Then
    3 KB (533 words) - 14:52, 29 October 2023
  • ...d cone has horizontal bases with radii <math>18</math> and <math>2</math>. A sphere is tangent to the top, bottom, and lateral surface of the truncated <math>\mathrm{(A)}\ 6
    3 KB (520 words) - 19:12, 20 November 2023
  • {{AMC10 Problems|year=2008|ab=A}} ...}\ {\small\text{AM}}</math> the machine has completed one third of the day's job. At what time will the doughnut machine complete the job?
    14 KB (2,138 words) - 15:08, 18 February 2023
  • ...th>. Circles <math>B</math> and <math>C</math> are congruent. Circle <math>A</math> has radius <math>1</math> and passes through the center of <math>D</ pair A=(-1,0),B=(2/3,8/9),C=(2/3,-8/9),D=(0,0);
    5 KB (785 words) - 00:29, 31 July 2023
  • A basketball player made <math>5</math> baskets during a game. Each basket was worth either <math>2</math> or <math>3</math> points. <math>\textbf{(A)}\ 2 \qquad \textbf{(B)}\ 3 \qquad \textbf{(C)}\ 4 \qquad \textbf{(D)}\ 5 \
    14 KB (2,199 words) - 13:43, 28 August 2020
  • ...B=11, BC=5, CD=19,</math> and <math>DA=7</math>. Bisectors of <math>\angle A</math> and <math>\angle D</math> meet at <math>P</math>, and bisectors of < <math>\textbf{(A)}\ 28\sqrt{3}\qquad \textbf{(B)}\ 30\sqrt{3}\qquad \textbf{(C)}\ 32\sqrt{3}
    12 KB (2,015 words) - 20:54, 9 October 2022
  • Of the students attending a school party, <math>60\%</math> of the students are girls, and <math>40\%</ ...l whole numbers of kilometers per hour. Find the sum of the squares of Ed's biking, jogging, and swimming rates.
    9 KB (1,536 words) - 00:46, 26 August 2023
  • ...te which is three-quarters the rate that Rudolph bikes, but Jennifer takes a five-minute break at the end of every two miles. Jennifer and Rudolph begin ...by <math>14</math> cm. Ten slices are cut from the cheese. Each slice has a width of <math>1</math> cm and is cut parallel to one face of the cheese. T
    7 KB (1,167 words) - 21:33, 12 August 2020
  • ...math>B</math>, and <math>AABAA</math>, while <math>BBAB</math> is not such a sequence. How many such sequences have length 14? ...math>n-2</math> ending in a <math>B</math>. Thus, we have the [[recursion]]s
    7 KB (1,173 words) - 22:39, 28 November 2023
  • ...a</math>. Point <math>P</math> is the foot of the perpendicular from <math>A</math> to line <math>CT</math>. Suppose <math>\overline{AB} = 18</math>, an ...- (9,0), A= O + (9,0), C=A+(18,0), T = 9 * expi(-1.2309594), P = foot(A,C,T);
    8 KB (1,333 words) - 00:18, 1 February 2024
  • &\text {ii. all ten digits, a through j are all distinct.}\\ &\text {iii.} a + b + c + d + e + f + g + h + i + j = 45\end{align*}</cmath>
    6 KB (909 words) - 07:27, 12 October 2022
  • ...to arrange these plates for dinner around her circular table if she doesn't want the 2 green plates to be adjacent? ...1+i)^{17} - (1-i)^{17}</math>, where <math>i=\sqrt{-1}</math>. Find <math>|S|</math>.
    6 KB (992 words) - 14:15, 13 February 2018
  • ...mark at exactly the same time. How many minutes has it taken them?<!-- don't remove the following tag, for PoTW on the Wiki front page--></onlyinclude> ...</math> be the time Rudolph takes disregarding breaks and <math>\frac{4}{3}r</math> be the time Jennifer takes disregarding breaks. We have the equation
    3 KB (432 words) - 11:53, 19 June 2019
  • Let <math>r</math>, <math>s</math>, and <math>t</math> be the three roots of the equation Find <math>(r + s)^3 + (s + t)^3 + (t + r)^3</math>.
    7 KB (1,251 words) - 19:18, 2 January 2024
  • ...h> units in the positive <math>x</math>-direction. Given that the particle's position after <math>150</math> moves is <math>(p,q)</math>, find the great ...the x-axis. If <math>(x', y')</math> is the position of the particle after a move from <math>P</math>, then we have two equations for <math>x'</math> an
    5 KB (725 words) - 22:37, 28 January 2024
  • real r=44-6*35^.5; pair A=(0,96),B=(-28,0),C=(28,0),X=C-(64/3,0),Y=B+(4*r/3,0),P=X+(0,16),Q=Y+(0,r),M=foot(Q,X,P);
    6 KB (1,065 words) - 20:12, 9 August 2022
  • pair C=(0,1), A=(1,6), B=(4,1), D=(1.5,2); draw(D--A--B--C--D--B);
    7 KB (1,221 words) - 18:57, 3 July 2013
  • <math>\mathrm{(A)}\ 120\qquad\mathrm{(B)}\ 130\qquad\mathrm{(C)}\ 140\qquad\mathrm{(D)}\ 150 ...d as <math>I</math>, and set Han's speed as <math>H</math>. Therefore, Jan's speed is <math>H+5.</math>
    6 KB (1,033 words) - 15:19, 1 July 2021
  • <cmath> k_0 k_1 \dotsc k_n = a^2+a+1, </cmath> ...on <math>n</math>. For our base case, <math>n=0</math>, we may let <math>a</math> be positive integer.
    11 KB (1,964 words) - 03:38, 17 August 2019
  • ...<math>y</math> and write <math>y - x</math> in its place. Prove that after a finite number of such operations, we can end up with at least one <math>0</ ...gers <math>A = \left < a_1, a_2, a_3 \right ></math> so that <math>R \cdot A = 0</math> continues to hold. (For example, if we replace <math>r_1</math>
    2 KB (407 words) - 23:37, 21 July 2020
  • (''Gabriel Carroll'') Let <math>n</math> be a positive integer. Denote by <math>S_n</math> the set of points <math>(x, y) .../math> paths (a partition of <math>S_n</math> into <math>m</math> paths is a set <math>\mathcal{P}</math> of <math>m</math> nonempty paths such that eac
    9 KB (1,585 words) - 01:00, 14 August 2014
  • ...eory of [[finite]] [[group]]s. They give a partial converse to [[Lagrange's Theorem]], and are one of the most important results in the field. They ar :'''Theorem.''' Every finite group contains a [[Sylow p-subgroup |Sylow <math>p</math>-subgroup]].
    11 KB (2,071 words) - 12:25, 9 April 2019
  • ...be the intersection of <math>CF</math> and <math>AD</math>. Then, '''Routh's Theorem''' states that <cmath>[GHI]=\dfrac{(rst-1)^2}{(rs+r+1)(st+s+1)(tr+t+1)}[ABC]</cmath>
    2 KB (267 words) - 00:02, 24 March 2021
  • A category, <math>\mathcal{C}</math>, is a mathematical object consisting of: * A [[class]], <math>\text{Ob}(\mathcal{C})</math> of objects.
    5 KB (792 words) - 19:01, 7 April 2012
  • ...s as a Renewable Energy Engineer for the Southern Company, and Hannah runs a lab at Jupiter Falls University where she researches biomass (renewable fue When the Kubiks went on vacation to San Diego last year, they spent a day at the San Diego Zoo.
    71 KB (11,749 words) - 01:31, 2 November 2023
  • ...^{2002} </math>. Prove that it is possible to color every subset of <math>S </math> either blue or red so that the following conditions hold: (a) the union of any two red subsets is red;
    3 KB (486 words) - 09:21, 14 May 2021
  • ...</math> arc of circle B. What is the ratio of circle A's area and circle B's area? <math>\textbf{(A)}\ 4/9 \qquad \textbf{(B)}\ 2/3 \qquad \textbf{(C)}\ 5/6 \qquad \textbf{(D)
    3 KB (492 words) - 14:46, 31 January 2024
  • <math>\textbf{(A)}\ 45 \qquad \textbf{(B)}\ 48 \qquad \textbf{(C)}\ 50 \qquad \textbf{(D)}\ ...in, we find <math>d=12</math>. From <math>d=rt</math>, we find that <math>r</math>, our answer, is <math>\boxed{\textbf{(B) }48 }</math>.
    2 KB (396 words) - 00:19, 1 February 2024
  • <math>\textbf{(A)} ~99 \qquad\textbf{(B)} ~100 \qquad\textbf{(C)} ~101 \qquad\textbf{(D)} ~1 ...an area of <math>\frac{\pi}{\sqrt{2}}</math> is <math>r</math>. Find <math>r^{2}</math>
    11 KB (1,695 words) - 14:33, 7 March 2022
  • Let <math>A = (a_{ij})(i, j = 1, 2, \cdots, n)</math> be a square matrix whose elements are non-negative integers. Suppose that whenev ...ecome smaller. So we have<cmath> T \ge (n - S)^2 + S^2 \ge 2(\frac{n - S + S}{2})^2 = \frac{n^2}{2}</cmath>by AM - QM inequality.
    6 KB (1,192 words) - 14:14, 29 January 2021
  • ...ns were <math>A</math> and <math>B</math>, respectively. Each polygon had a side length of <math>2</math>. Which of the following is true? ...B\qquad \textbf{(D)}\ A </math> <math>= \frac {7}{5}B\qquad \textbf{(E)}\ A = \frac {49}{25}B</math>
    4 KB (630 words) - 21:27, 30 December 2023
  • A circle with center <math>C</math> is tangent to the positive <math>x</math> <math>\textbf{(A)}\ 3 \qquad \textbf{(B)}\ 4 \qquad \textbf{(C)}\ 6 \qquad \textbf{(D)}\ 8 \
    2 KB (314 words) - 21:07, 16 January 2020
  • ...tly five chips, three red and two white. Chips are randomly removed one at a time without replacement until all the red chips are drawn or all the white \textbf{(A) }\frac {3}{10}
    2 KB (423 words) - 12:21, 11 June 2022
  • ...d pairs of real numbers <math>(a,b)</math> such that <math>(a+bi)^{2002} = a-bi</math>. \text{(A) }1001
    4 KB (743 words) - 19:54, 14 March 2024
  • ...micircle but outside the circle is shaded. What fraction of the semicircle's area is shaded? \mathrm{(A)}\ \frac{1}{2}
    2 KB (380 words) - 09:21, 8 June 2021
  • ...h> pyramids and a new polyhedron <math>R</math>. How many edges does <math>R</math> have? <math>\mathrm{(A)}\ 200\qquad
    9 KB (1,567 words) - 13:43, 19 August 2023
  • ...nt muffin or a <math>75</math>-cent bagel. Her total cost for the week was a whole number of dollars. How many bagels did she buy? <math>\text{(A) } 1\qquad\text{(B) } 2\qquad\text{(C) } 3\qquad\text{(D) } 4\qquad\text{(E
    15 KB (2,262 words) - 00:53, 18 June 2021
  • real r=5, R=6; path t=r*dir(0)--r*dir(20)--R*dir(20)--R*dir(0);
    3 KB (546 words) - 04:26, 16 January 2023
  • ...s a complex number <math>z</math> with imaginary part <math>164</math> and a positive integer <math>n</math> such that Let <math>z = a + 164i</math>.
    5 KB (850 words) - 12:53, 23 December 2023
  • ...cle having <math>\overline{CD}</math> as a diameter. Let <math>I</math> be a point outside <math>\triangle ABC</math> such that <math>\overline{AI}</mat ...h>. Hence <math>AQ = 144, BP = 1225, AB = 1369</math> and the radius <math>r = OD = 210</math>.
    12 KB (1,970 words) - 22:53, 22 January 2024
  • '''Cauchy's Integral Formula''' is a fundamental result in [[complex analysis]]. It states that if <math>U</math> is a subset of
    4 KB (689 words) - 17:19, 18 January 2024
  • ...ng a blue stripe, a red stripe, a white stripe, and a pink stripe. Pink is a mixture of red and white, not necessarily in equal amounts. When Bill finis ...ath>b</math>, and <math>c</math> are positive real numbers such that <math>a^{\log_3 7} = 27</math>, <math>b^{\log_7 11} = 49</math>, and <math>c^{\log_
    8 KB (1,366 words) - 21:33, 3 January 2021
  • <math>\mathrm{(A)}\ 15\qquad\mathrm{(B)}\ 16\qquad\mathrm{(C)}\ 17\qquad\mathrm{(D)}\ 21\qqu <math>\text{(A)}\ -2 \qquad \text{(B)}\ -1 \qquad \text{(C)}\ 0 \qquad \text{(D)}\ 1 \qqua
    15 KB (2,165 words) - 03:32, 13 April 2024
  • ...n should peruse this material carefully, as it may contain the keys to one's future success. ...rtains to that situation. I hope after you read this text, you will become a better math student, armed with another tool to solve difficult problems. B
    10 KB (1,797 words) - 02:05, 24 October 2023
  • ...<math>t</math> is not divisible by the square of any prime. Find <math>r+s+t</math>. ...c{3}{5}</math>, point <math>B</math> lies between <math>M</math> and <math>A</math> on the semicircular arc. We will first compute the length of <math>\
    11 KB (1,849 words) - 19:43, 2 January 2023
  • <math>\text{(A)}\ 947 \qquad \text{(B)}\ 1037 \qquad \text{(C)}\ 1047 \qquad \text{(D)}\ 1 <math>\text{(A)}\ 1 \qquad \text{(B)}\ 2 \qquad \text{(C)}\ 3 \qquad \text{(D)}\ 4 \qquad
    15 KB (2,059 words) - 15:03, 6 October 2021
  • ...ext{C}</math>. If this sum equals the sum of the dates behind <math>\text{A}</math> and <math>\text{B}</math>, then the letter is for(int a=0; a<6; ++a)
    1 KB (231 words) - 10:51, 8 September 2019
  • Bases. These confuse a lot of us. Hopefully this will make it better. ...as a subscript. Every number in base 10 (our normal numbers) does not have a subscript.
    3 KB (552 words) - 20:57, 9 July 2009
  • ...}</math> and <math>\overline{BC}</math> at points <math>R</math> and <math>S</math>, respectively. Prove that quadrilateral <math>PQRS</math> is cyclic We will use directed angles in this solution. Extend <math>QR</math> to <math>T</math> as follows:
    6 KB (973 words) - 19:24, 18 October 2018
  • ...r <math>r</math> such that <math>(s_i - s_j)r</math> and <math>(t_i - t_j)/r</math> are integers for all <math>i</math> and <math>j</math>. ...ac{c_2}{d_2} - \frac{c_1}{d_1})</math> is an integer. Now we can set <math>r = \frac{b_1 b_2}{d_1 d_2}</math>, because the least common denominator of <
    2 KB (463 words) - 12:19, 21 August 2020
  • A '''semisimple module''' is, informally, a [[module]] that is not far removed from [[simple module]]s.
    6 KB (1,183 words) - 15:02, 18 August 2009
  • ...2k\}</math> whose base 2 representation has precisely three <math>1</math>s. * (a) Prove that, for each positive integer <math>m</math>, there exists at leas
    7 KB (1,298 words) - 19:59, 9 February 2024
  • ...er and over and over again. Thus, the olympiad package is a lifesaver. Let's look at this simple bit of code: (Note, the import olympiad command is not real r=2.5, a=3.5;
    1 KB (209 words) - 01:48, 7 January 2010
  • Which pair of numbers does NOT have a product equal to <math>36</math>? <math>\text{(A)}\ \{ -4,-9\} \qquad \text{(B)}\ \{ -3,-12\} \qquad \text{(C)}\ \left\{ \df
    14 KB (1,797 words) - 11:13, 28 December 2022
  • ...and the top portion is a sphere that holds 100,000 liters of water. Logan's miniature water tower holds 0.1 liters. How tall, in meters, should Logan m <math>\textbf{(A)}\ 0.04 \qquad \textbf{(B)}\ \frac{0.4}{\pi} \qquad \textbf{(C)}\ 0.4 \qqua
    1 KB (198 words) - 18:08, 28 June 2021
  • <math>\textbf{(A)}\ 4 \qquad \textbf{(B)}\ 5 \qquad \textbf{(C)}\ 6 \qquad \textbf{(D)}\ 7 \ ...gative, all <math>3</math> zeros must be double roots because the function's degree is <math>6</math>.
    5 KB (969 words) - 21:33, 22 June 2022
  • {{AMC10 Problems|year=2010|ab=A}} Mary’s top book shelf holds five books with the following widths, in centimeters:
    13 KB (1,902 words) - 11:20, 5 March 2023
  • ...[operator]] refers to the [[polynomial]] whose roots are the [[eigenvalue]]s of the operator. It carries much information about the operator. ...det(tI - A)</math>, which is a <math>n</math>th degree polynomial in <math>t</math>. Here, <math>I</math> refers to the <math>n\times n</math> [[identit
    19 KB (3,412 words) - 14:57, 21 September 2022
  • ...h>p</math> be the probability that exactly one of the selected divisors is a perfect square. The probability <math>p</math> can be expressed in the form ...mes 99 \times 999 \times \cdots \times \underbrace{99\cdots9}_{\text{999 9's}}</math> is divided by <math>1000</math>.
    8 KB (1,243 words) - 21:58, 10 August 2020
  • .../math>, respectively. Line <math>\ell</math> divides region <math>\mathcal{R}</math> into two regions with areas in the ratio <math>1: 2</math>. Suppose label("$A$",(-16.43287,-9.3374),NE/2);
    10 KB (1,418 words) - 23:05, 20 October 2021
  • ...an be easily proved through strong induction. Starting from 2010, which is a multiple of 15, we must first purge 1 lemming. We can then purge 4 lemmings ...atisfies <math>b_1+b_2+\ldots+b_{10}\equiv 0 \pmod{3}</math>, there exists a corresponding <math>(a_1, a_2, \ldots, a_{10})</math> such that <math>a_1+a
    36 KB (6,214 words) - 20:22, 13 July 2023
  • ...math> denote the distance from <math>P</math> to the closest side of <math>S</math>. The probability that <math>\frac{1}{5}\le d(P)\le\frac{1}{3}</math> ...where <math>a</math> and <math>b</math> are integers satisfying <math>1\le a < b \le 20</math>. Find the greatest positive integer <math>n</math> such t
    8 KB (1,246 words) - 21:58, 10 August 2020
  • ...<math>r</math> is not divisible by the square of any prime. Find <math>p+q+r</math>. ...o use Power of a Point on <math>BA</math> and <math>CD</math>. By Power of a Point Theorem, <math>CP\cdot PD=1\cdot 2=BP\cdot PA</math>. Since <math>BP+
    10 KB (1,507 words) - 00:31, 19 November 2023
  • ...2w-4</math>, where <math>i^2=-1</math>. Find <math>|a+b+c|</math>.<!-- don't remove the following tag, for PoTW on the Wiki front page--></onlyinclude> == Solution (vieta's) ==
    4 KB (735 words) - 02:41, 27 June 2022
  • ...econd-degree [[polynomial]]s <math>f(x)</math> with integer [[coefficient]]s and integer zeros for which <math>f(0)=2010</math>. ...guishable. In one case, <math>(a,r,s) = (2010,1,1)</math>, we have <math>r=s</math>. The other <math>80</math> cases are double counting, so there are <
    6 KB (972 words) - 16:01, 16 September 2023
  • Let <math>ABCD</math> be a convex quadrilateral such that diagonals <math>AC</math> and <math>BD</math pair A,B,C,D;
    2 KB (446 words) - 08:09, 10 April 2023
  • ...Find with proof the smallest <math>n</math> such that <math>P(n)</math> is a multiple of <math>2010</math>. ...s. We claim that the relation <math>R = \{(j, k)\in [n]\times[n]\mid jk\in S\}</math> is an equivalence relation on <math>[n]</math>.
    12 KB (2,338 words) - 20:30, 13 February 2024
  • Let <math>AXYZB</math> be a convex pentagon inscribed in a semicircle of diameter <math>AB</math>. Denote by <math>P, Q, R, S</math> the feet of the perpendiculars from <math>Y</math> onto
    13 KB (2,178 words) - 14:14, 11 September 2021
  • ...(\text{i})\quad</math> For which positive integers <math>n</math> is there a finite set <math>S_n</math> of <math>n</math> distinct positive integers su ...sitive integers such that the geometric mean of any finite subset of <math>S</math> is an integer?
    4 KB (855 words) - 03:12, 23 October 2022
  • A high school basketball game between the Raiders and Wildcats was tied at th <math>\textbf{(A)}\ 30 \qquad \textbf{(B)}\ 31 \qquad \textbf{(C)}\ 32 \qquad \textbf{(D)}\
    6 KB (990 words) - 19:14, 25 March 2024
  • Let <math>a > 0</math>, and let <math>P(x)</math> be a polynomial with integer coefficients such that <math>P(1) = P(3) = P(5) = P(7) = a</math>,
    7 KB (1,166 words) - 00:04, 26 February 2024
  • ...perpendicular to <math>EF</math> meets <math>\omega</math> again at <math>R</math>. Line <math>AR</math> meets ω again at <math>P</math>. The circumci ...t lines <math>DI</math> and <math>PQ</math> meet on the line through <math>A</math> perpendicular to <math>AI</math>.
    5 KB (792 words) - 01:52, 19 November 2023
  • <math>\text{(A)}\ 2 \qquad \text{(B)}\ 3 \qquad \text{(C)}\ 4 \qquad \text{(D)}\ 5 \qquad <math>\text{(A)}\ \text{Jose} \qquad \text{(B)}\ \text{Thuy} \qquad \text{(C)}\ \text{Kare
    13 KB (1,880 words) - 13:35, 19 February 2020
  • This page is intended to serve as a brief, beginners reference to the [[Python]] programming language. In Python, there is no need to specify a type when storing information in a variable. It is basic and straightforward to store information.
    28 KB (4,762 words) - 21:20, 12 June 2023
  • <math> \textbf{(A)}\ 10\qquad\textbf{(B)}\ 15\qquad\textbf{(C)}\ 16\qquad\textbf{(D)}\ 17\qqu ...answers 13 questions correctly, answers 7 questions incorrectly and doesn't answer the last 5. What is his score?
    16 KB (2,215 words) - 19:18, 10 April 2024
  • <math>\text{(A)}\ \dfrac{6}{x} \qquad \text{(B)}\ \dfrac{6}{x+1} \qquad \text{(C)}\ \dfrac ...\text{b}\cdot \text{c}</math>, what is the value of <math>\begin{tabular}{r|l}3&4 \\ \hline 1&2\end{tabular}</math>?
    14 KB (1,920 words) - 19:31, 31 January 2024
  • <math>\textbf{(A)}\ 1930 \qquad \textbf{(B)}\ 1931 \qquad \textbf{(C)}\ 1932 \qquad \textbf{ ...ions such that the red bag is empty is equal to <math>2^7</math>, since it's equivalent to distributing the <math>7</math> candies into <math>2</math> b
    4 KB (750 words) - 22:59, 18 January 2024
  • ...<math>A.</math> How many hours did it take the power boat to go from <math>A</math> to <math>B</math>? \textbf{(A)}\ 3 \qquad
    5 KB (928 words) - 23:01, 14 April 2022
  • <math> \textbf{(A)}\ 5\qquad <math> \textbf{(A)}\ y - x\qquad \textbf{(B)}\ x - y\qquad \textbf{(C)}\ \frac {y - x}{xy}\qq
    25 KB (3,872 words) - 14:21, 20 February 2020
  • {{AMC10 Problems|year=2011|ab=A}} A cell phone plan costs <math>\textdollar 20</math> each month, plus <math>5<
    13 KB (1,903 words) - 18:09, 19 April 2021
  • ...<math>c</math>, define <math>\boxed{a,b,c}</math> to mean <math>a^b-b^c+c^a</math>. Then <math>\boxed{1,-1,2}</math> equals <math>\text{(A)} \ -4 \qquad \text{(B)} \ -2 \qquad \text{(C)} \ 0 \qquad \text{(D)} \ 2 \
    20 KB (2,814 words) - 08:15, 27 June 2021
  • ...tersection of the lines <math>AF</math> and <math>BC</math>, and let <math>T</math> be the point of intersection of the lines <math>AG</math> and <math> .../math>. Since <math>AK</math> and <math>AL</math> are tangents from <math>A</math> to the circle <math>J</math>, <math>AK = AL</math>. Then, we have <
    7 KB (1,189 words) - 01:22, 19 November 2023
  • ...olution in jar <math>C</math> is added to jar B. At the end both jar <math>A</math> and jar <math>B</math> contain solutions that are <math>50\%</math> .../math> the <math>y</math>-axis. In the new coordinate system, point <math>A</math> is on the positive <math>x</math>-axis, and point <math>B</math> is
    10 KB (1,634 words) - 22:21, 28 December 2023
  • ...e the sum of the elements in <math>R</math>. Find the remainder when <math>S</math> is divided by 1000. ...<math>2^i \equiv 2^j \pmod{1000}</math>. All that is left is to find <math>S</math> in mod <math>1000</math>. After some computation:
    11 KB (1,668 words) - 22:10, 24 February 2023
  • ..., <math>s</math>, and <math>t</math> are positive integers. Find <math>r+s+t</math>. ...h>a^2+b^2+c^2=1</math>. The distance from a point <math>(X,Y,Z)</math> to a plane with equation <math>Ax+By+Cz+D=0</math> is
    4 KB (613 words) - 11:37, 8 September 2022
  • ...14 intersect at point <math>P</math>. The distance between the [[midpoint]]s of the two chords is 12. The quantity <math>OP^2</math> can be represented The line through the midpoint of a chord of a circle and the center of that circle is perpendicular to that chord, so <ma
    11 KB (1,720 words) - 03:12, 18 December 2023
  • <math>\text{(A)} \div \qquad \text{(B)}\ \times \qquad \text{(C)} + \qquad \text{(D)}\ - \ What is the degree measure of the smaller angle formed by the hands of a clock at 10 o clock?
    17 KB (2,394 words) - 19:51, 8 May 2023
  • ...r of opposite sides are parallel. The internal angles satisfy <math>\angle A = 3\angle D</math>, <math>\angle C = 3\angle F</math>, and <math>\angle E = ...B = \beta</math>, <math>AB=DE=p</math>, <math>BC=EF=q</math>, <math>CD=FA=r</math>. Define the vectors: <cmath>\vec{u} = \vec{AB} + \vec{DE}</cmath> <
    11 KB (1,925 words) - 12:07, 31 August 2023
  • ...front page--><onlyinclude>Points <math>R</math>, <math>S</math> and <math>T</math> are vertices of an equilateral triangle, and points <math>X</math>, drawn using any three of these six points as vertices?<!-- don't remove the following tag, for PoTW on the Wiki front page--></onlyinclude>
    3 KB (549 words) - 16:35, 23 July 2023
  • Isabella's house has <math>3</math> bedrooms. Each bedroom is <math>12</math> feet lon <math>\textbf{(A) } 678 \qquad\textbf{(B) } 768 \qquad\textbf{(C) } 786 \qquad\textbf{(D) }
    15 KB (2,297 words) - 12:57, 19 February 2020
  • Here is a list of '''Olympiad Books''' that have Olympiad-level problems used to trai ...om/community/c142h405377 here] about these books or request new books. Let's categorize books into '''Theory''' books, '''Problem''' books, and '''Both'
    17 KB (2,261 words) - 00:30, 22 April 2024
  • ...an actual [[AMC]] (American Mathematics Competitions 8, 10, or 12) exam. A number of '''Mock AMC''' competitions have been hosted on the [[Art of Prob ...r more information on Mock AMCs, as well as tips on how to write them, and a complete list of all Mock AMC 10s and Mock AMC 12s.
    18 KB (2,206 words) - 19:41, 24 December 2020
  • ...umber <math>5b9</math>. If <math>5b9</math> is divisible by 9, then <math>a+b</math> equals <math> \text{(A)}\ 2\qquad\text{(B)}\ 4\qquad\text{(C)}\ 6\qquad\text{(D)}\ 8\qquad\text{(E
    20 KB (3,108 words) - 14:14, 20 February 2020
  • Jamie counted the number of edges of a cube, Jimmy counted the numbers of corners, and Judy counted the number of <math>\mathrm{(A)}\ 12 \qquad\mathrm{(B)}\ 16 \qquad\mathrm{(C)}\ 20 \qquad\mathrm{(D)}\ 22
    16 KB (2,236 words) - 12:02, 19 February 2024
  • = Part A: Each correct answer is worth 5 points = <math>\text{(A)}\ 1 \qquad \text{(B)}\ 2 \qquad \text{(C)}\ 3 \qquad \text{(D)}\ 4 \qquad
    16 KB (2,317 words) - 03:54, 24 October 2014
  • Bridget bought a bag of apples at the grocery store. She gave half of the apples to Ann. The <math> \textbf{(A)}\ 3\qquad\textbf{(B)}\ 4\qquad\textbf{(C)}\ 7\qquad\textbf{(D)}\ 11\qquad\
    18 KB (2,551 words) - 18:46, 27 February 2024
  • ...sector of <math>\angle A</math>, find all possible values for <math>\angle A</math>. ...<math>X</math>, then <math>x\cdot r(x)</math> and <math>(x + (1 - x) \cdot r(x) )</math> both belong to <math>X</math>.
    2 KB (356 words) - 18:42, 18 July 2016
  • ...t</math>. Show that <math>k=a^2-3b\ge 0</math> and that <math>\sqrt k\le r-t</math>. ...ath>X</math>. Show that for any map <math>f: P\mapsto X</math> we can find a 10-element subset <math>Y</math> of <math>X</math>, such that <math>f(Y-\{k
    2 KB (319 words) - 16:33, 5 February 2018
  • ...<math>P</math>, <math>Q</math>, <math>R</math>, <math>S</math>, and <math>T</math> represent numbers located on the number line as shown. label("$-3$",(-3,-0.1),S);
    3 KB (506 words) - 00:24, 5 July 2013
  • ...<math>Q</math> has coordinates (2,2). What are the coordinates for <math>T</math> so that the area of triangle <math>PQT</math> equals the area of squ pair O,P,Q,R,T;
    2 KB (257 words) - 11:20, 22 March 2015
  • == Division A == ...math> such that <math>4^x = x^4</math>. Find the nearest integer to <math>S</math>.
    22 KB (3,694 words) - 23:58, 3 June 2022
  • ...<math>r,s,</math> and <math>t</math> is <math>1.</math> What is <math>r+s+t</math>? <math>\textbf{(A) } 7 \qquad \textbf{(B) } 9 \qquad \textbf{(C) } 12 \qquad \textbf{(D) } 15
    8 KB (1,386 words) - 15:10, 8 October 2023
  • ...math> beats <math>R</math>, <math>Q</math> beats <math>S</math>, and <math>T</math> finishes after <math>P</math> and before <math>Q</math>. Who could ...and }S \qquad \text{(D)}\ P\text{ and }T \qquad \text{(E)}\ P,S\text{ and }T</math>
    1 KB (228 words) - 00:12, 5 July 2013
  • In this addition problem, each letter stands for a different digit. ...sep}{0.5mm}\begin{array}{cccc}&T & W & O\\ +&T & W & O\\ \hline F& O & U & R\end{array} </math>
    1 KB (177 words) - 19:56, 15 April 2023
  • ...east one of her friends. How many classmates will not be invited to Sarah's party? pair a=(102,256), b=(68,131), c=(162,101), d=(134,150);
    2 KB (250 words) - 22:17, 5 January 2024
  • <math> \textbf{(A)}\ 5\frac{1}{3}\qquad\textbf{(B)}\ 11\qquad\textbf{(C)}\ 10\frac{2}{3}\qqua ...>. When <math>S=8</math>, <math>R=16</math>. When <math>S=10</math>, <math>R</math> is equal to:
    22 KB (3,306 words) - 19:50, 3 May 2023
  • '''Algebra A''' Let <math>x_1,x_2,\dots,x_n</math> be a sequence of integers, such that <math>-1\leq x_i\leq 2</math>, for <math>i=
    25 KB (4,154 words) - 16:27, 2 September 2011
  • ...students in Mr. Newton's class, and <math>9</math> students in Mrs. Young's class taking the AMC 8 this year. How many mathematics students at Euclid M <math> \textbf{(A)}\ 26 \qquad\textbf{(B)}\ 27\qquad\textbf{(C)}\ 28\qquad\textbf{(D)}\ 29\qq
    18 KB (2,768 words) - 21:05, 9 January 2024
  • ...math> apples at a cost of <math> 50 </math> cents per apple. She paid with a 5-dollar bill. How much change did Margie receive? <math>\textbf{(A) }\ \textdollar 1.50 \qquad \textbf{(B) }\ \textdollar 2.00 \qquad \textbf{
    16 KB (2,371 words) - 17:34, 9 January 2024
  • ...ution, one can write <math>x_n=P_n(x_1)</math> , where <math>P_n</math> is a polynomial with non-negative coefficients and zero constant term. Thus, <ma (a) <math>\displaystyle x_n<x_{n+1} \Leftrightarrow x_n>1-\frac{1}{n} \Leftrig
    9 KB (1,788 words) - 00:02, 30 January 2021
  • ...e the positive integer <math>n</math> such that <math>n^2+20n+40</math> is a perfect square. (Proposed by djmathman) ...umber of ordered integer pairs <math>(a,b)</math> such that <math>a^2+b^2|(a+b)^2</math>. (Proposed by djmathman)
    15 KB (2,444 words) - 21:46, 1 January 2012
  • <math> \textbf{(A)}\ 7\qquad\textbf{(B)}\ 8\qquad\textbf{(C)}\ 9\qquad\textbf{(D)}\ 10\qquad\ ...0</math>. Then the set of points <math>(x,y,z)</math> is a tetrahedron, or a triangular pyramid. The point <math>(x,y,z)</math> distributes uniformly in
    13 KB (2,133 words) - 01:22, 6 February 2024
  • ...each of the centers of the corresponding circles is among the vertices of a regular hexagon of side 2. What is the area enclosed by the curve? <math>\textbf{(A)}\ 2\pi+6\qquad\textbf{(B)}\ 2\pi+4\sqrt{3}\qquad\textbf{(C)}\ 3\pi+4\qquad
    5 KB (775 words) - 22:33, 22 October 2023
  • <math> \textbf{(A)}\ 5\qquad\textbf{(B)}\ \sqrt{26}\qquad\textbf{(C)}\ 3\sqrt{3}\qquad\textbf label("$Z$",Z,S);
    9 KB (1,496 words) - 02:40, 2 October 2022
  • ...igcup_{j=1}^{k}p_{j} </math>. The intersection of <math>P</math> and <math>S</math> consists of the union of all segments joining the midpoints of every <math> \textbf{(A)}\ 8\qquad\textbf{(B)}\ 12\qquad\textbf{(C)}\ 20\qquad\textbf{(D)}\ 23\qqua
    5 KB (940 words) - 17:13, 4 April 2020
  • A triangle is inscribed in a circle. The vertices of the triangle divide the circle into three arcs of l <math> \mathrm{(A) \ 6 } \qquad \mathrm{(B) \frac{18}{\pi^2} } \qquad \mathrm{(C) \frac{9}{\p
    4 KB (649 words) - 10:04, 20 May 2021
  • ...s with three not necessarily distinct digits, <math>abc</math>, with <math>a \neq 0</math> and <math>c \neq 0</math> such that both <math>abc</math> and ...e, and they have been traveling for <math>t</math> minutes. Find <math>n + t</math>.
    10 KB (1,617 words) - 14:49, 2 June 2023
  • ...math>b,</math> and <math>c</math> in the complex plane are the vertices of a right triangle with hypotenuse <math>h.</math> Find <math>h^2.</math> ...9}\cdot\frac{x^2+y^2}{4}=\frac{x^2}{9}+\frac{y^2}{9}</math>. Hence, <math>|a|^2+|b|^2+|c|^2=\frac{6x^2+6y^2}{9}=\frac{2x^2+2y^2}{3}=250</math>. Therefor
    12 KB (2,228 words) - 23:59, 20 January 2024
  • ...les <math>\omega</math> and <math>\gamma</math> meet at <math>E</math> and a second point <math>F</math>. Then <math>AF^2 = \frac mn</math>, where <math ...math>. Solving gives <math>x^2=\frac{49}{19}</math>, from which by Ptolemy's we see <math>AF=\frac{30}{\sqrt{19}}</math>. We conclude the answer is <mat
    13 KB (2,298 words) - 12:56, 10 September 2023
  • <math>\textbf{(A) }2+0+1+7\qquad\textbf{(B) }2 \times 0 +1+7\qquad\textbf{(C) }2+0 \times 1 Alicia, Brenda, and Colby were the candidates in a recent election for student president. The pie chart below shows how the vo
    12 KB (1,771 words) - 21:13, 20 January 2024
  • ...<math>q</math> and <math>r</math> are relatively prime. What is <math>p+q+r</math>? A permutation of the numbers <math>\{2</math>, <math>3</math>, <math>4</math>
    7 KB (1,309 words) - 11:13, 8 April 2012
  • ...e expressed in the form <math>\dfrac{r+\sqrt{s}}{t}</math>. Find <math>r+s+t</math>. ...rth power, the desired answer is <math>\dfrac{7+\sqrt{45}}{2} \implies r+s+t=\boxed{054}</math>.
    3 KB (543 words) - 18:51, 7 May 2020
  • We say that a finite set <math>\mathcal{S}</math> in the plane is <i> balanced </i> ...any two different points <math>A</math>, <math>B</math> in <math>\mathcal{S}</math>, there is
    4 KB (773 words) - 08:14, 19 July 2016
  • Eight card players are seated around a table. One remarks that at some moment, any player and his two neighbours h Prove that any real number <math>0<x<1</math> can be written as a difference of two positive and less than <math>1</math> irrational numbers.
    10 KB (1,695 words) - 10:03, 10 May 2012
  • Note that, first, you need to make sure you have a version of JVM(Java Virtual Machine) installed. See [[Install Java]] page f You can read if you have a 32- or 64-bit system under System Type:
    8 KB (1,222 words) - 17:47, 9 October 2014
  • ...t</math>. Show that <math>k=a^2-3b\ge 0</math> and that <math>\sqrt k\le r-t</math>. By Vieta's Formulas, <math>a=-r-s-t</math>, <math>b=rs+st+rt</math>, and <math>c=-rst</math>.
    3 KB (674 words) - 18:17, 13 June 2017
  • ...ta IAB</math>, <math>\Delta IBC</math>, and <math>\Delta ICA</math> lie on a circle whose center is the circumcenter of <math>\Delta ABC</math>. ...>, and <math>O_b</math>, respectively. It then suffices to show that <math>A</math>, <math>B</math>, <math>C</math>, <math>O_a</math>, <math>O_b</math>,
    3 KB (504 words) - 19:25, 14 October 2021
  • <math> \mathrm{(A)\ } 4x+3y=xy \qquad \mathrm{(B) \ }y=\frac{4x}{6-y} \qquad \mathrm{(C) \ } <math> \mathrm{(A)\ } -\frac{h}{3} \qquad \mathrm{(B) \ }\frac{h}{3} \qquad \mathrm{(C) \ }
    15 KB (2,151 words) - 14:04, 19 February 2020
  • ...t of tangency. If <math> TP=4 </math> and <math> T'Q=9 </math> then <math> r </math> is pair O = (0,0); pair T = dir(90);
    2 KB (337 words) - 12:44, 5 July 2013
  • Given a cube, determine the ratio of the volume of the octahedron formed by connect ...ath> and <math> X </math> is on the same side of line <math> AB </math> as a given point <math> C </math>.
    5 KB (837 words) - 12:22, 27 May 2012
  • ...equation <math> 4y''+3y'-y=0 </math> and its solution <math> y=e^{\lambda t} </math>, what are the values of <math> \lambda </math>? ...orn is a point on the ground, and that the elven rope is so light it makes a straight line from the unicorn to the tower.
    3 KB (525 words) - 13:59, 27 May 2012
  • ....4), M=(1,-4.2), N=(1,-4), P=(1,-3.8), Q=(1,-3.6), R=(1,-3.4), S=(1,-3.2), T=(1,-3), U=(1,-2.8), V=(1,-2.6), W=(1,-2.4), Z=(1,-2.2), E_1=(1.4,-2.6), F_1 ...D(B--(8,-2),linewidth(0.4)); D(B--S_1); D(T--R_1); D(N--Q_1); D(H--P_1); D(A--O_1); D(C_1--E_1); D(E_1--F_1); D(F_1--D_1); D(D_1--C_1); D(G_1--L_2); D(L
    5 KB (847 words) - 19:59, 5 February 2023
  • If the radius of a circle is a rational number, its area is given by a number which is: ...ational} \qquad \textbf{(C)\ } \text{integral} \qquad \textbf{(D)\ } \text{a perfect square }\qquad \textbf{(E)\ } \text{none of these} </math>
    23 KB (3,556 words) - 15:35, 30 December 2023
  • <math>\textbf{(A)}\ y^2-5\sqrt{y^2-25} \qquad \textbf{(B)}\ -y^2 \qquad \textbf{(C)}\ y^2 \\ <math>\textbf{(A)}\ 4 \text{ and }1 \qquad \textbf{(B)}\ \text{only }1 \qquad \textbf{(C)}\
    23 KB (3,535 words) - 16:29, 24 April 2020
  • <math> \textbf{(A)}\ 3.75\times 10^{-7}\qquad\textbf{(B)}\ 3\frac{3}{4}\times 10^{-7}\qquad\t The smaller angle between the hands of a clock at <math>12:25</math> p.m. is:
    22 KB (3,509 words) - 21:29, 31 December 2023
  • ...er <math>k</math> (as a function of <math>n</math>) such that there exists a convex <math>n</math>-gon <math>A_{1}A_{2}\dots A_{n}</math> for which exac pair A, B, C, D, E, F, G, H, I, J, K, L, M, N, O, P, Q, R, S, T, U;
    5 KB (871 words) - 18:59, 10 May 2023
  • (a) every square that does not contain a checker shares a side with one that does; ...ven squares, such that every two consecutive squares of the sequence share a side.
    4 KB (747 words) - 05:23, 13 May 2023
  • Here are the problems from the 2020 Mock Combo AMC 10 II, a mock contest created by the AoPS user fidgetboss_4000. ...friend, Sara, along with eight other classmates, are randomly seated along a row of ten chairs. What is the probability that Fred sits next to Sara?
    15 KB (2,452 words) - 03:03, 4 July 2020
  • {{AMC12 Problems|year=2013|ab=A}} <math>\textbf{(A)} \ 4 \qquad \textbf{(B)} \ 5 \qquad \textbf{(C)} \ 6 \qquad \textbf{(D)} \
    14 KB (2,206 words) - 19:31, 15 May 2024
  • {{AMC10 Problems|year=2013|ab=A}} A taxi ride costs \$1.50 plus \$0.25 per mile traveled. How much does a 5-mile taxi ride cost?
    12 KB (1,894 words) - 15:59, 3 January 2024
  • ...ath>, <math>b</math>, and <math>c</math> are positive integers. Find <math>a+b+c</math>. ...3]{9}+1</math> to rationalize the denominator, and we therefore have <math>r=\frac{\sqrt[3]{81}+\sqrt[3]{9}+1}{8}</math>, and the answer is <math>\boxed
    3 KB (573 words) - 21:11, 27 August 2023
  • ...\text{B}</math>, and <math>\text{C}</math> are digits. Find <math>100\text{A}+10\text{B}+\text{C}</math>. Positive integers <math>a</math> and <math>b</math> satisfy the condition
    8 KB (1,402 words) - 12:17, 13 March 2020
  • ..., and <math>c</math> are integers. Find the number of polynomials in <math>S</math> such that each of its roots <math>z</math> satisfies either <math>|z ...real and two nonreal roots which are conjugates. This follows from [[Vieta's formulas]].
    8 KB (1,393 words) - 19:00, 24 May 2023
  • In triangle <math>ABC</math>, points <math>P,Q,R</math> lie on sides <math>BC,CA,AB</math> respectively. Let <math>\omega_A pair A = (-1.0, 3.0);
    14 KB (1,830 words) - 18:22, 10 May 2023
  • ...integers that do not exceed <math>4</math>. Find the remainder when <math>T</math> is divided by <math>1000</math>. Draw in the diagonals of a regular octagon. What is the sum of all distinct angle measures, in degree
    7 KB (1,173 words) - 21:04, 7 December 2018
  • ...of 24, the hexagon has the same area as the original rectangle. Find <math>a^2</math>. pair A,B,C,D,E,F,R,S,T,X,Y,Z;
    3 KB (454 words) - 22:00, 24 January 2024
  • ...0</math>. Find the sum of all possible values of <math>|b|</math>.<!-- don't remove the following tag, for PoTW on the Wiki front page--></onlyinclude> ...</math> in <math>p(x)</math> and <math>q(x)</math> are both equal to <math>a</math>, and equating the two coefficients gives
    6 KB (1,069 words) - 15:51, 31 December 2023
  • {{AMC10 Problems|year=2016|ab=A}} <math>\textbf{(A)}\ 99\qquad\textbf{(B)}\ 100\qquad\textbf{(C)}\ 110\qquad\textbf{(D)}\ 121\
    14 KB (2,104 words) - 22:26, 16 September 2022
  • ...nteger <math>n</math> and all real <math>t, x, y</math> <cmath>P(tx, ty) = t^nP(x, y)</cmath> (that is, <math>P</math> is homogeneous of degree <math>n< (ii) for all real <math>a, b, c</math>, <cmath>P(b + c, a) + P(c + a, b) + P(a + b, c) = 0,</cmath>
    3 KB (563 words) - 16:21, 29 January 2021
  • ...c37</math>. Find <math>S</math> when <math>R = \sqrt {48}</math> and <math>T = \sqrt {75}</math>. <math> \textbf{(A)}\ 28\qquad\textbf{(B)}\ 30\qquad\textbf{(C)}\ 40\qquad\textbf{(D)}\ 42\qqu
    969 bytes (143 words) - 22:19, 10 April 2023
  • ...decagon (<math>12</math> sides) is inscribed in a circle with radius <math>r</math> inches. The area of the dodecagon, in square inches, is: ...bf{(B)}\ 2r^2\qquad\textbf{(C)}\ \frac{3r^2\sqrt{3}}{4}\qquad\textbf{(D)}\ r^2\sqrt{3}\qquad\textbf{(E)}\ 3r^2\sqrt{3} </math>
    902 bytes (147 words) - 22:17, 3 October 2014
  • <math>\textbf{(A)}\ \frac{4y-1}{8} \qquad <math>\textbf{(A)}\ \frac{\sqrt{3}}{6} \qquad
    17 KB (2,459 words) - 22:40, 10 April 2023
  • <math>\textbf{(A)}\ 1 \qquad \textbf{(B)}\ 2 \qquad \textbf{(C)}\ 3 \qquad \textbf{(D)}\ 4 \ ...lf-pound packages for just \$3 per package." What is the regular price for a full pound of fish, in dollars? (Assume that there are no deals for bulk)
    15 KB (2,162 words) - 20:05, 8 May 2023
  • ...mill on 3 days this week. He went 2 miles each day. On Monday he jogged at a speed of 5 miles per hour. He walked at the rate of 3 miles per hour on Wed <math>\textbf{(A)}\ 1 \qquad \textbf{(B)}\ 2 \qquad \textbf{(C)}\ 3 \qquad \textbf{(D)}\ 4 \
    2 KB (323 words) - 17:57, 5 May 2022
  • <math> \textbf{(A)}\ \sqrt{2}\qquad\textbf{(B)}\ 2\qquad\textbf{(C)}\ 4\qquad\textbf{(D)}\ 8\ <math> \textbf{(A)}\ \sqrt{3}\qquad\textbf{(B)}\ \sqrt{5}\qquad\textbf{(C)}\ 3\qquad\textbf{(
    17 KB (2,633 words) - 15:44, 16 September 2023
  • Walking down Jane Street, Ralph passed four houses in a row, each painted a different color. He passed the orange house before the red house, and he pa <math>\textbf{(A)}\ 2\qquad\textbf{(B)}\ 3\qquad\textbf{(C)}\ 4\qquad\textbf{(D)}\ 5\qquad\t
    3 KB (532 words) - 17:28, 5 May 2024
  • ...th> is a square, <math>P</math>, <math>Q</math>, <math>R</math>, and <math>S</math> are midpoints of the sides <math>AB</math>, <math>BC</math>, <math>C label("A",(0,2),NW);
    1 KB (158 words) - 19:59, 30 January 2023
  • ...1P_2P_3EP_4P_5</math> be a heptagon. A frog starts jumping at vertex <math>S</math>. From any vertex of the heptagon except <math>E</math>, the frog may ...f left/right (L/R) paths of length <math>\le 11</math> that start at <math>S</math> and end at either <math>P_4</math> or <math>P_3</math>.
    4 KB (649 words) - 17:29, 22 December 2023
  • If <math>2</math> is a solution (root) of <math>x^3+hx+10=0</math>, then <math>h</math> equals: <math>\textbf{(A)}10\qquad
    21 KB (3,242 words) - 21:27, 30 December 2020
  • ...d have the same number of pennies and nickels. In cents, how much are Leah's coins worth? <math> \textbf{(A)}\ 33\qquad\textbf{(B)}\ 35\qquad\textbf{(C)}\ 37\qquad\textbf{(D)}\ 39\qqu
    13 KB (2,066 words) - 14:08, 1 November 2022
  • ...ld have the same number of pennies as nickels. In cents, how much are Leah's coins worth? <math> \textbf {(A) } 33 \qquad \textbf {(B) } 35 \qquad \textbf {(C) } 37 \qquad \textbf {(D)
    13 KB (2,011 words) - 21:54, 8 November 2022
  • A sphere is inscribed in a truncated right circular cone as shown. The volume of the truncated cone is <math>\text{(A) } \dfrac32 \quad \text{(B) } \dfrac{1+\sqrt5}2 \quad \text{(C) } \sqrt3 \q
    6 KB (1,060 words) - 19:15, 11 August 2023
  • A sphere is inscribed in a truncated right circular cone as shown. The volume of the truncated cone is real r=(3+sqrt(5))/2;
    4 KB (703 words) - 16:24, 9 September 2022
  • ...s Abe. Abe begins to paint the room and works alone for the first hour and a half. Then Bea joins Abe, and they work together until half the room is pai ...has none of the three risk factors given that he does not have risk factor A is <math>\frac{p}{q}</math>, where <math>p</math> and <math>q</math> are re
    8 KB (1,410 words) - 00:04, 29 December 2021
  • Jon and Steve ride their bicycles along a path that parallels two side-by-side train tracks running the east/west dir ...arly, the second train has to cover a distance equal to its own length, at a rate of <math>r_2 + \dfrac{1}{3}</math>. Since the times are equal and <mat
    5 KB (784 words) - 13:59, 30 November 2021
  • ...nts <math>\overline{EG}</math> and <math>\overline{FH}</math> intersect at a point <math>P</math>, and the areas of the quadrilaterals <math>AEPH, BFPE, pair A = (0,sqrt(850));
    9 KB (1,404 words) - 21:07, 13 October 2023
  • ...is a positive integer not divisible by the square of any prime. Find <math>a+b+c</math>. ...=90^\circ</math>. But <math>DF=FE</math>, so <math>\triangle DEF</math> is a 45-45-90 triangle. Letting <math>DG=3x</math>, we have that <math>EG=4x</ma
    10 KB (1,643 words) - 22:30, 28 January 2024
  • A chord which is the perpendicular bisector of a radius of length 12 in a circle, has length <math> \textbf{(A)}\ 3\sqrt3\qquad\textbf{(B)}\ 27\qquad\textbf{(C)}\ 6\sqrt3\qquad\textbf{(D
    18 KB (2,788 words) - 13:55, 20 February 2020
  • ...n, as <math>m</math> is made arbitrarily close to zero, the value of <math>r</math> is: <math>\text{(A) arbitrarily close to } 0\quad\\
    2 KB (399 words) - 13:19, 27 July 2018
  • <math>\frac{a}{b},a \ne b,b \ne 0</math>, the value of the fraction is changed to <math>\frac{c <math>\text{(A) } \frac{1}{c-d}\quad
    16 KB (2,662 words) - 14:12, 20 February 2020
  • ...if they traveled in the same direction, but would pass each other in <math>t</math> hours if they traveled in opposite directions. The ratio of the spee <math>\text{(A) } \frac {r + t}{r - t} \qquad
    2 KB (370 words) - 01:27, 31 January 2023
  • <math>\textbf{(A)}\ 2y \qquad <math>\textbf{(A)}\ y = \frac{1}{3} x + 8 \qquad
    17 KB (2,512 words) - 18:30, 12 October 2023
  • <math> ABCD</math> is a rectangle (see the accompanying diagram) with <math> P</math> any point on MP("A",(-2,1),NW);MP("B",(2,1),NE);MP("C",(2,-1),SE);MP("D",(-2,-1),SW);
    2 KB (368 words) - 14:07, 28 August 2022
  • <math>\text{(A)}\ x \qquad <math>\textbf{(A)}\ \text{ equiangular}\qquad
    15 KB (2,412 words) - 05:09, 27 November 2020
  • <math>\text{(A) I and II}\qquad If a dealer could get his goods for <math>8</math>% less while keeping his selli
    17 KB (2,732 words) - 13:54, 20 February 2020
  • ...Prove that <math>\gamma_1, \gamma_2</math>, and <math>\gamma_3</math> have a second tangent in common. ...ath>K_2</math> is the intersection of the inversion circle <math>(B, r_B = a)</math> with the line <math>AB</math>.
    5 KB (904 words) - 13:42, 29 January 2021
  • ...1's in a row and randomly writes + or - between each pair of consecutive 1's. <math>\text{(A) }0\qquad
    2 KB (247 words) - 03:34, 14 June 2018
  • <math>\text{(A) } 0 \quad \text{(R) } 17 \quad
    929 bytes (137 words) - 22:05, 10 January 2019
  • Ted's favorite number is equal to Find the remainder when Ted’s favorite number is divided by 25.
    2 KB (270 words) - 14:35, 29 July 2018
  • <math>\text{(A) } 0\quad \text{(R) } -8\qquad
    1 KB (202 words) - 16:48, 24 November 2018
  • ...such that <math>2008N</math> is a perfect square and <math>2007N</math> is a perfect cube. <math>\text{(A) }0 \quad
    2 KB (340 words) - 19:49, 30 June 2018
  • Let <math>n</math> be a positive integer. Consider <cmath>S=\{(x,y,z)~:~x,y,z\in \{0,1,\ldots,n \},~x+y+z>0\}</cmath>
    3 KB (506 words) - 13:38, 23 March 2024
  • ...ar from <math>A</math> to <math>BD</math>. Points <math>S</math> and <math>T</math> lie on sides <math>AB</math> and <math>AD</math>, respectively, such pair A = (0,1), C=(0,-1);
    6 KB (1,071 words) - 03:58, 8 September 2018
  • <math>\textbf{(A)}\ \log_{10}50 \qquad <math>\textbf{(A)}\ \text{a parabola} \qquad
    19 KB (2,907 words) - 14:16, 20 February 2020
  • <math>\textbf{(A)}\ (0,0)\qquad <math>\textbf{(A)}\ -14 \qquad
    20 KB (3,122 words) - 14:17, 20 February 2020
  • <math>\textbf{(A)}\ \frac{1}{25} \qquad An automobile travels <math>a/6</math> feet in <math>r</math> seconds. If this rate is maintained for <math>3</math> minutes, how
    18 KB (2,905 words) - 18:33, 5 April 2023
  • ...lines representing the altitudes, medians, and interior angle bisectors of a triangle that is isosceles, but not equilateral, is: <math>\textbf{(A)}\ 9\qquad \textbf{(B)}\ 7\qquad \textbf{(C)}\ 6\qquad \textbf{(D)}\ 5\qqua
    26 KB (3,950 words) - 21:09, 31 August 2020
  • ...equal to <math>4^3</math>, so <math>64</math> is both a perfect square and a perfect cube. (a) Find the smallest positive integer multiple of <math>12</math> that is a perfect square.
    11 KB (1,648 words) - 09:55, 20 December 2021
  • pair P,R; R=unit(C1-D1)+D1;
    5 KB (846 words) - 22:10, 7 November 2014
  • Find a <math>3</math>-digit integer less than <math>200</math> where each digit is pair A,B,C,D;
    5 KB (826 words) - 22:12, 7 November 2014
  • Suppose <math>r, s</math>, and <math>t</math> are three different positive integers and that their product is <mat What is the smallest possible value of the sum <math>r+s+t</math>?
    550 bytes (83 words) - 02:46, 13 January 2019
  • ...sum <math>S= 1+ \frac{1}{3}+\frac{1}{3^2}+ \frac{1}{3^3}+ \cdots</math> as a reduced fraction. (b) Express the infinite sum <math>T=\frac{1}{5}+ \frac{1}{25}+ \frac{2}{125}+ \frac{3}{625}+ \frac{5}{3125}+ \c
    1 KB (156 words) - 05:26, 12 January 2019
  • <math>\text{(A)}\ 18 \qquad \text{(B)}\ 20 \qquad \text{(C)}\ 16 \qquad \text{(D)}\ 6 \qqu label("$S$",(0,0),SW);
    2 KB (349 words) - 02:41, 23 October 2014
  • ...P, AQ</math> and <math>AR</math> are the perpendiculars dropped from <math>A</math> onto <math>CD, CB</math> extended and <math>DE</math> extended, ...62), P=(C+D)/2, Q=C+3.10*dir(C--B), R=D+3.10*dir(D--E), S=C+4.0*dir(C--B), T=D+4.0*dir(D--E);
    4 KB (702 words) - 17:13, 17 April 2020
  • {{AMC12 Problems|year=2015|ab=A}} <math> \textbf{(A)}\ -125\qquad\textbf{(B)}\ -120\qquad\textbf{(C)}\ \frac{1}{5}\qquad\textbf
    13 KB (2,117 words) - 12:33, 24 August 2023
  • ...ax+2a</math> are integers. What is the sum of the possible values of <math>a</math>? <math>\textbf{(A)}\ 7 \qquad\textbf{(B)}\ 8 \qquad\textbf{(C)}\ 16 \qquad\textbf{(D)}\ 17 \q
    7 KB (1,193 words) - 14:25, 25 July 2022
  • ...Furthermore, <math>AB = CD = 39</math> and <math>PQ = 48</math>. Let <math>R</math> be the midpoint of <math>\overline{PQ}</math>. What is <math>\overli <math>\textbf{(A)}\; 180 \qquad\textbf{(B)}\; 184 \qquad\textbf{(C)}\; 188 \qquad\textbf{(D)
    7 KB (1,188 words) - 19:18, 19 October 2023
  • ...math>n</math>. Find the smallest positive integer satisfying <math>s(n) = s(n+864) = 20</math>. ...it has to have 3 digits, because if it had only two, the maximum of <math>s(n)</math> is 18.
    10 KB (1,797 words) - 14:06, 21 January 2024
  • Let <math>f(x)</math> be a third-degree polynomial with real coefficients satisfying Since <math>f(x)</math> is a third degree polynomial, it can have at most two bends in it where it goes
    8 KB (1,474 words) - 10:00, 10 November 2023
  • ...math> on <math>\overline{AC}</math>, and vertices <math>R</math> and <math>S</math> on <math>\overline{BC}</math>. In terms of the side length <math>PQ ...ath> is half the area of the triangle. Using Heron's formula, since <math>s = \frac{12 + 17 + 25}{2} = 27</math>,
    7 KB (1,180 words) - 14:08, 14 February 2023
  • ...and <math>1</math> satisfy <math>S(a)S(-a)=2016</math>. Find <math>S(a)+S(-a)</math>. ...th> is directly proportional to <math>k</math>. The probability of rolling a <math>7</math> with this pair of dice is <math>\frac{m}{n}</math>, where <m
    8 KB (1,360 words) - 12:19, 29 January 2022
  • A quadrilateral is inscribed in a circle of radius <math>200\sqrt{2}</math>. Three of the sides of this quadr <math>\textbf{(A) }200\qquad \textbf{(B) }200\sqrt{2}\qquad\textbf{(C) }200\sqrt{3}\qquad\te
    24 KB (3,861 words) - 14:17, 26 April 2024
  • <math>\textbf{(A)}\ 15\qquad\textbf{(B)}\ 16\qquad\textbf{(C)}\ 24\qquad\textbf{(D)}\ 27\qqu ...,2),(1,2),(0,2)</math>. Thus our answer is <math>5\times 3=\boxed{\textbf{(A) }15}.</math>
    7 KB (1,135 words) - 09:28, 5 September 2022
  • ...n segment <math>\overline{PQ}</math>, show that <math>M</math> moves along a circle. <cmath>A=(1,0) P=(1-a,b), Q=(1-a,-b)</cmath>,
    5 KB (902 words) - 09:58, 20 August 2021
  • ...n segment <math>\overline{PQ}</math>, show that <math>M</math> moves along a circle. pair A=(1,0);
    4 KB (760 words) - 16:45, 29 April 2020
  • The AoPS font is a font that can be used to make certain special symbols in the forum. You enc | <span class="aops-font">a</span>
    6 KB (924 words) - 21:31, 22 February 2024
  • ...points are designated on <math>\triangle ABC</math>: the 3 vertices <math>A</math>, <math>B</math>, and <math>C</math>; <math>3</math> other points on ...der is always the positive integer <math>s \neq r</math>. Find <math>m+n+r+s</math>.
    7 KB (1,163 words) - 16:43, 2 June 2022
  • ...ex had the least. The three numbers of peanuts that each person had formed a geometric progression. Alex eats <math>5</math> of his peanuts, Betty eats ...solving for <math>r</math> gives <math>r=\dfrac{4}{3}</math>, since <math>r>1</math>, so Alex had <math>\dfrac{3}{4} \cdot 144=\boxed{108}</math> peanu
    5 KB (788 words) - 02:50, 1 March 2024
  • ...mial <math>x^3-ax^2+bx-a</math> are real. In fact, for this value of <math>a</math> the value of <math>b</math> is unique. What is this value of <math>b <math>\textbf{(A)}\ 8\qquad\textbf{(B)}\ 9\qquad\textbf{(C)}\ 10\qquad\textbf{(D)}\ 11\qquad
    3 KB (568 words) - 17:14, 20 April 2024
  • ...> and <math>b</math> are relatively prime positive integers. What is <math>a + b?</math> (For example, he succeeds if his sequence of tosses is <math>HT <math>\textbf{(A)}\ 69\qquad\textbf{(B)}\ 151\qquad\textbf{(C)}\ 257\qquad\textbf{(D)}\ 293\
    5 KB (881 words) - 13:34, 7 October 2023
  • ...<math>\sqrt{STARS} = SAT</math>, what are <math>A, R, S</math>, and <math>T</math>? (a) Determine <math>f\circ g(x)</math> and <math>g\circ f(x)</math>.
    5 KB (846 words) - 03:36, 19 January 2019
  • ...the series is <math>1</math>. What is the smallest possible value of <math>S?</math> <math>\textbf{(A)}\ \frac{1+\sqrt{5}}{2} \qquad
    7 KB (1,264 words) - 14:57, 5 July 2022
  • A strictly increasing sequence of positive integers <math>a_1</math>, <math>a We first create a similar sequence where <math>a_1=1</math> and <math>a_2=2</math>. Continuin
    5 KB (883 words) - 15:24, 6 January 2024
  • Triangle <math>ABC</math> has <math>AB=40,AC=31,</math> and <math>\sin{A}=\frac{1}{5}</math>. This triangle is inscribed in rectangle <math>AQRS</ma ...by drawing triangle ABC, where <math>A</math> is obtuse. Therefore, angle A is acute. Let angle <math>CAS=n</math> and angle <math>BAQ=m</math>. Then,
    9 KB (1,526 words) - 02:31, 29 December 2021
  • Since all the terms of the sequences are integers, and 100 isn't very big, we should just try out the possibilities for <math>b_2</math>. Wh We have <math>a_k=r^{k-1}</math> and <math>b_k=(k-1)d</math>. First, <math>b_{k-1}<c_{k-1}=100<
    6 KB (983 words) - 01:18, 2 February 2023
  • <math>\mathrm{(A)}\, 8</math> <math>\mathrm{(A)}\, 15\quad\mathrm{(B)}\, \frac{40}{3}</math>
    31 KB (4,811 words) - 00:02, 4 November 2023
  • ...ath>, <math>PQ</math> extended to <math>D</math> and <math>\angle n</math> a right angle, then: path anglemark2(pair A, pair B, pair C, real t=8, bool flip=false)
    3 KB (560 words) - 10:11, 3 May 2020
  • Find all functions <math>f:\mathbb{R}\rightarrow \mathbb{R}</math> such that for all real numbers <math>x</math> and <math>y</math>, < ...that <math>f(t) = 0 \implies f(-t) = 0</math> for all <math>t \in \mathbb{R}.</math>
    8 KB (1,391 words) - 00:13, 28 February 2020
  • ...</math> are colinear. This can be proved by the Trigonometric Form of Ceva's Theorem for <math>\triangle I_AI_BI_C.</math> We find point <math>T</math> on line <math>YZ,</math> we prove that <math>TY \perp OI_A</math> an
    6 KB (998 words) - 21:36, 17 October 2022
  • ...and <math>\omega</math>. Prove that <math>B</math>, <math>E</math>, <math>R</math> are collinear. ...th>ABCD</math> is harmonic, hence the tangents at <math>C</math> and <math>A</math> concur on <math>BD</math> at <math>X</math>, say.
    5 KB (927 words) - 21:28, 28 May 2016
  • Assume that <math> P\ne\{3\}</math> and let's try to determine the second smallest element <math> q = \min (P\setminus\{3 ...> n</math>, a non-sense). Therefore, <math> n = 3^t n'</math> where <math> t = 0</math> or <math> 1</math> and <math> n'</math> does not have prime divi
    6 KB (999 words) - 18:15, 7 August 2021
  • ...<math>X</math>, then <math>x\cdot r(x)</math> and <math>(x + (1 - x) \cdot r(x) )</math> both belong to <math>X</math>. ...>s(x)</math> are distinct elements of <math>X</math>, then <math>r(x) \neq s(x)</math> for any <math>0 < x < 1</math>.
    2 KB (357 words) - 16:49, 18 May 2018
  • ...\sigma(U)}{|S|} = \pi(S) </cmath> for all integers <math>\, m \geq \sigma(S)</math>. ...<math>\left|\bigcup_{k \in U}A_k\right| = \sum_{\emptyset \neq U \subseteq S} (-1)^{|U|+1}\left|\bigcap_{k \in U}A_k\right|</math>.
    3 KB (628 words) - 10:42, 4 August 2023
  • ...h>3</math>, <math>4</math>, and <math>5</math> are each used once to write a five-digit number <math>PQRST</math>. The three-digit number <math>PQR</mat <math>\textbf{(A) }1\qquad\textbf{(B) }2\qquad\textbf{(C) }3\qquad\textbf{(D) }4\qquad \text
    5 KB (781 words) - 22:06, 21 January 2024
  • ...can distribute 1 piece of candy among 243 hungry schoolchildren sitting in a row -3. We have a triangle abc with sides ab, bc, ca with lengths α, β, and γ, respectivel
    14 KB (2,904 words) - 18:24, 16 May 2017
  • {{AMC10 Problems|year=2017|ab=A}} <math>\textbf{(A)}\ 70\qquad\textbf{(B)}\ 97\qquad\textbf{(C)}\ 127\qquad\textbf{(D)}\ 159\q
    15 KB (2,285 words) - 18:02, 28 October 2023
  • ...ertex of the square coincides with the right-angle vertex of the triangle. A square with side length <math>y</math> is inscribed in another right triang <math>\textbf{(A) } \dfrac{12}{13} \qquad \textbf{(B) } \dfrac{35}{37} \qquad \textbf{(C) }
    3 KB (461 words) - 22:24, 8 January 2023
  • Distinct points <math>P</math>, <math>Q</math>, <math>R</math>, <math>S</math> lie on the circle <math>x^{2}+y^{2}=25</math> and have integer coord <math>\textbf{(A) } 3 \qquad \textbf{(B) } 5 \qquad \textbf{(C) } 3\sqrt{5} \qquad \textbf{(
    5 KB (864 words) - 01:42, 17 August 2023
  • The vertices <math>V</math> of a centrally symmetric hexagon in the complex plane are given by <math>\textbf{(A) } \dfrac{5\cdot11}{3^{10}} \qquad \textbf{(B) } \dfrac{5^2\cdot11}{2\cdot3
    18 KB (2,878 words) - 01:47, 16 December 2023
  • ...ertex of the square coincides with the right-angle vertex of the triangle. A square with side length <math>y</math> is inscribed in another right triang <math> \textbf{(A)}\ \frac{12}{13}
    2 KB (351 words) - 14:55, 10 June 2023
  • ...ly tangent to each other and internally tangent to the largest semicircle. A circle centered at <math>P</math> is drawn externally tangent to the two sm <math> \textbf{(A)}\ \frac{3}{4}
    13 KB (1,982 words) - 17:12, 20 December 2022
  • ...reen. Two paintings that can be obtained from one another by a rotation or a reflection of the entire figure are considered the same. How many different pair A, B, C, D, E, F;
    7 KB (1,057 words) - 23:27, 27 August 2022
  • ...laced in an urn---one green, one red, and two white. The players each draw a ball at random without replacement. Whoever gets the green ball gives one c <math>\textbf{(A)}\quad \dfrac{7}{576} \qquad \qquad \textbf{(B)}\quad \dfrac{5}{192} \qquad
    12 KB (1,935 words) - 11:37, 16 December 2023
  • pair C=(0,0),B=(0,2*sqrt(3)),A=(5,0); real t = .385, s = 3.5*t-1;
    22 KB (3,622 words) - 17:11, 6 January 2024
  • ...he triangle is closer to vertex <math>B</math> than to either vertex <math>A</math> or vertex <math>C</math> can be written as <math>\frac{p}{q}</math>, pair A,B,C,D,X,Z,P;
    9 KB (1,411 words) - 22:18, 29 January 2024
  • ...unterclockwise, and writing the color of the points in that order - either R or B. For example, possible sequences include <math>RB</math>, <math>RBBR</ ...h> from my sequence when I choose them. I define this to be a move. Hence, a possible move sequence of <math>BBRRRB</math> is:
    8 KB (1,465 words) - 15:30, 12 June 2020
  • <math>\textbf{(A) }5 \qquad \textbf{(B) }4 + \sqrt{7} + \sqrt{10} \qquad \textbf{(C) } 10 \q ...^2-11^2} \cdot \frac{(70-11)(70+11)}{(100-7)(100+7)}</cmath><math>\textbf{(A) } 1 \qquad \textbf{(B) } \frac{9951}{9950} \qquad \textbf{(C) } \frac{4780
    16 KB (2,417 words) - 01:03, 28 April 2022
  • ...th>\overarc{TR}</math> and <math>\overarc{SR}</math> are each one-sixth of a circle with radius 2. What is the area of the region shown? ...abel("$S$", (1,1.732), W); label("$T$", (3,1.732), E); label("$R$", (2,0), S);</asy>
    4 KB (711 words) - 21:00, 16 January 2024
  • <math>\begin{cases}x^y=y^x\\y=ax\end{cases}~\mbox{where }a\neq1\mbox{ and }a>0</math> ...> to <math>A</math> and from <math>P</math> to <math>B</math> are given by a constant <math>k</math>. Justify your answer discussing every possibility f
    2 KB (377 words) - 14:52, 7 January 2018
  • ...0<\gcd(d, a)<100</math>. Which of the following must be a divisor of <math>a</math>? <math>\textbf{(A)} \text{ 5} \qquad \textbf{(B)} \text{ 7} \qquad \textbf{(C)} \text{ 11} \q
    10 KB (1,599 words) - 04:51, 6 August 2023
  • ...o form a convex regular polygon, three of whose vertices are labeled <math>A,B,</math> and <math>C</math>. What is the least possible area of <math>\tri <math>\textbf{(A) } \frac{1}{6}\sqrt{6} \qquad \textbf{(B) } \frac{3}{2}\sqrt{2}-\frac{3}{2}
    11 KB (1,708 words) - 12:01, 18 March 2023
  • ...t) linear factors with integer coefficients. Find the remainder when <math>S</math> is divided by <math>1000</math>. Then, <math>a=c+d</math> and <math>b=cd</math>.
    10 KB (1,670 words) - 16:38, 15 January 2024
  • ...y once in every <math>2 \times 2</math> subtable.The sum of all numbers of a regular table is called the total sum of the table. With any four numbers, In a regular table either each row contains exactly two of the numbers or each c
    5 KB (799 words) - 01:20, 12 January 2023
  • ...gn*}bc\sin\frac A2\sin\frac B2\sin\frac C2&=bc\cdot\dfrac{(a-b+c)(b-c+a)(c-a+b)}{8abc}\\&=\dfrac{(30-32+34)(32-34+30)(34-30+32)}{8\cdot 32}=\boxed{126}. ...minimize the area for <math>[A I_1 I_2]</math> because <math>b, c, \angle A, \angle B, \angle C</math> are all constants while only <math>\sin \alpha</
    13 KB (2,200 words) - 21:36, 6 January 2024
  • ...t random among all subsets of <math>U</math>, the probability that <math>s(T)</math> is divisible by <math>3</math> is <math>\frac{m}{n}</math>, where < The question asks us for the probability that a randomly chosen subset of the set of the first 18 positive integers has the
    26 KB (4,044 words) - 13:58, 24 January 2024
  • ...tions are from the "Ultimate Question" of the [[2007 iTest]], but with the T-values substituted. This is for people who want to work on the problems wi Let <math>R</math> be the region consisting of points <math>(x,y)</math> of the Cartesi
    4 KB (636 words) - 00:41, 25 June 2018
  • ...math> such that there exists a number <math>A</math> such that <math>a_n = A</math> for infinitely many values of <math>n</math>. Let <math>\mathbb{R}</math> be the set of real numbers , determine all functions
    4 KB (720 words) - 12:12, 5 August 2021
  • One night, over dinner Jerry poses a challenge to his younger children: "Suppose we travel <math>50</math> miles ...the beautiful landscape your mother likes to photograph from the passenger's seat, we
    2 KB (421 words) - 20:19, 6 August 2023
  • ...hords <math>MP</math>, <math>PQ</math>, and <math>NR</math> are each <math>s</math> units long and chord <math>MN</math> is <math>d</math> units long. label("1", (5,0), S);
    7 KB (1,181 words) - 20:41, 17 April 2022
  • Line <math>\ell</math> passes through <math>A</math> and into the interior of the equilateral triangle <math>ABC</math>. real r = 31, t = -10;
    3 KB (488 words) - 12:54, 7 December 2018
  • ...square of any prime, and <math>\gcd(t,r,p)=1</math>. Compute <math>p+q+r+s+t</math>. Using the definition of a cosine, we have <math>XC = 8\sqrt{3} \cos(x)</math>. By the [[Law of Sines]
    3 KB (558 words) - 20:13, 4 January 2019
  • ...<math>AB=7</math>, <math>BC=9</math>, <math>CD=15</math>, and there exists a circle, lying inside the quadrilateral and having center <math>I</math>, th ...pposite sides of <math>QMPN</math> have equal length, <math>QMPN</math> is a [[parallelogram]]. Thus, the diagonals bisect each other, and <math>QI = IP
    3 KB (586 words) - 23:47, 8 January 2019
  • ...<math>\sqrt{STARS} = SAT</math>, what are <math>A, R, S</math>, and <math>T</math>? <math>A = 3, T = 9, S = 1</math> and <math>R = 2</math>
    2 KB (385 words) - 12:27, 21 August 2023
  • ...lvia, and Ted play the following game. Each starts with <math> \$1</math>. A bell rings every <math>15</math> seconds, at which time each of the players <math>\textbf{(A) } \frac{1}{7} \qquad\textbf{(B) } \frac{1}{4} \qquad\textbf{(C) } \frac{1}
    9 KB (1,568 words) - 11:15, 18 October 2023
  • ...h> for <math>k = 2</math>, <math>3</math>, <math>....</math> What is <math>a+b+c</math>? <math>\textbf{(A)} \; -6 \qquad \textbf{(B)} \; 0 \qquad \textbf{(C)} \; 6 \qquad \textbf{(D
    4 KB (731 words) - 22:18, 12 March 2023
  • <math>\textbf{(A) } \frac{5}{8} \qquad \textbf{(B) } \frac{4}{5} \qquad \textbf{(C) } \frac{ ...h>n-2</math> is prime." Which of the following values of <math>n</math> is a counterexample to this statement?
    15 KB (2,458 words) - 23:52, 12 November 2023
  • ...ath>\triangle ABC</math>. An ''excircle'' of <math>\triangle ABC</math> is a circle in the exterior of <math>\triangle ABC</math> that is tangent to one ...However, the sum of the radius of <math>\omega</math> and <math>\frac{rs}{s-b}</math> is equivalent to the distance between the incenter and the the <m
    21 KB (3,915 words) - 19:55, 10 October 2023
  • ...(z))</math> are the vertices of a right triangle in the complex plane with a right angle at <math>f(z)</math>. There are positive integers <math>m</math ...\frac{f(f(z))-f(z)}{z-f(z)}\in i\mathbb R .</cmath>However, <math>f(t)-t=t(t-20)</math>, so
    8 KB (1,534 words) - 22:17, 28 December 2023
  • ...point of <math>\Gamma</math> and <math>\ell</math> that is closer to <math>R</math>. Line <math>AJ</math> meets <math>\Omega</math> again at <math>K</ma We construct circle <math>\omega</math> centered at <math>R</math> which maps <math>\Gamma</math> into <math>\Gamma.</math>
    4 KB (681 words) - 01:41, 19 November 2023
  • = Pascal's Identity = Pascal's Identity states that
    15 KB (2,425 words) - 09:25, 13 February 2020
  • ...is welcomed to edit, BUT ONLY IN GOOD WAYS! The AoPS Secret Governemtn has a backup of this page. ...ng it. For example, can a butterfly that flaps its wings in Brazil trigger a tornado in Texas?
    35 KB (5,882 words) - 18:08, 28 June 2021
  • <math>\textbf{(A) }2\qquad \textbf{(R) }26\qquad
    4 KB (334 words) - 12:49, 15 January 2020
  • ...On the first day, it starts at point <math>O</math>, faces east, and walks a distance of <math>5</math> units due east. Each night the bug rotates <math pair O, A, B, C, D, F, G, H, I, P, X;
    9 KB (1,380 words) - 16:12, 2 January 2024
  • ...and <math>g(x)=x^2+cx+d.</math> Find the number of ordered triples <math>(a,b,c)</math> of integers with absolute values not exceeding <math>10</math> ...math>f(2),f(4)</math> are the roots of <math>g</math>. This means by Vieta's, that:
    6 KB (964 words) - 22:33, 18 January 2024
  • ...ath>p</math> are relatively prime positive integers, and <math>n</math> is a positive integer not divisible by the square of any prime. Find <math>m+n+p label("$A$", (-6.8002301023571095,8.267690318323321), NE * labelscalefactor);
    35 KB (5,215 words) - 23:08, 29 October 2023
  • ...ons, <math>x=3,4,a,b.</math> Find the sum of all possible values of <math>(a+b)^2.</math> ...ain. Now, it's easy to find that <math>a = -2.5, b = -3.5</math>, yielding a value of <math>36</math>. Finally, we add <math>49 + 36 = \boxed{085}</math
    10 KB (1,819 words) - 19:34, 17 September 2023
  • The following is a power of a point solution to this menace of a problem: var phi=75.5, theta=130, r=4.8;
    16 KB (2,678 words) - 22:45, 27 November 2023
  • pair A, B, C, D, E, F, P; A = 55*sqrt(3)/3 * dir(90);
    16 KB (2,592 words) - 15:40, 13 April 2024
  • ...oring a quadratic is to turn the quadratic into a product of 2 [[binomial]]s. ...the roots of the quadratic, and where <math>r+s=-b</math> and <math>r\cdot s=c</math>.
    2 KB (367 words) - 12:15, 15 July 2021
  • 4. The Infinity Numeral, PI, is the second deity of the Almighty Gmaas's heaven. 5. The Infinite Logarithm, E, is the third deity of the Almighty Gmaas's heaven.
    85 KB (13,971 words) - 18:02, 16 May 2024
  • ...ly. Find the perimeter of the triangle ABC which minimizes <math>\frac{s}{r}</math>. <cmath>r = \frac{x+696-y}{2} = 348 - a</cmath>
    4 KB (674 words) - 16:03, 25 February 2021
  • Ike and Mike go into a sandwich shop with a total of <math>\$30.00</math> to spend. Sandwiches cost <math>\$4.50</math> <math>\textbf{(A) }6\qquad\textbf{(B) }7\qquad\textbf{(C) }8\qquad\textbf{(D) }9\qquad\textb
    17 KB (2,585 words) - 15:27, 4 May 2024
  • <math>\textbf{(A) } 22 \qquad\textbf{(B) } 23 \qquad\textbf{(C) } 24 \qquad\textbf{(D) } 25 ...th>3 \left\lfloor \frac{999}{n} \right\rfloor \pm 1</math>, so it is never a multiple of <math>3</math>. Thus, we are looking for all <math>n \neq 1</ma
    12 KB (1,978 words) - 22:13, 29 September 2023
  • ...by another reflection across the <math>x</math>-axis will not return <math>T</math> to its original position.) <math>\textbf{(A) } 12 \qquad \textbf{(B) } 15 \qquad \textbf{(C) } 17 \qquad \textbf{(D) }
    8 KB (1,247 words) - 15:45, 23 September 2023
  • <math>\textbf{(A) } 12 \qquad\textbf{(B) } 15 \qquad\textbf{(C) } 18 \qquad\textbf{(D) } 21\ ...start bashing by plugging in numbers that satisfy these conditions: <math>a=4</math> is the first number that works so we get <math>\ell = 33</math>, <
    17 KB (2,544 words) - 12:09, 1 September 2023
  • ...P=1</math>, <math>BP=\sqrt{3}</math>, and <math>CP=2</math>. What is <math>s</math>? <math>\textbf{(A) } 1+\sqrt{2} \qquad \textbf{(B) } \sqrt{7} \qquad \textbf{(C) } \frac{8}{3
    16 KB (2,509 words) - 17:49, 8 February 2024
  • The number <math>a=\frac{p}{q}</math>, where <math>p</math> and <math>q</math> are relatively <cmath> \lfloor x \rfloor \cdot \{x\} = a \cdot x^2</cmath>
    9 KB (1,541 words) - 17:13, 21 July 2022
  • ...peration four times: he draws a ball from the urn at random and then takes a ball of the same color from the box and returns those two matching balls to <math>\textbf{(A) } \frac16 \qquad \textbf{(B) }\frac15 \qquad \textbf{(C) } \frac14 \qquad
    11 KB (1,928 words) - 22:40, 12 November 2023
  • As shown in the figure below, six semicircles lie in the interior of a regular hexagon with side length 2 so that the diameters of the semicircles <math>\textbf{(A) } 6\sqrt3 - 3\pi \qquad \textbf{(B) } \frac{9\sqrt3}{2} - 2\pi \qquad \tex
    17 KB (2,392 words) - 12:36, 24 December 2023
  • ...th> be a chord in the circle that intersects <math>\overline{AB}</math> at a point <math>E</math> such that <math>BE=2\sqrt5</math> and <math>\angle AEC <math>\textbf{(A)}\ 96 \qquad\textbf{(B)}\ 98 \qquad\textbf{(C)}\ 44\sqrt5 \qquad\textbf{(D
    10 KB (1,599 words) - 10:05, 6 October 2023
  • {{AMC10 Problems|year=2021|ab=A}} What is the value of<cmath>(2^2-2)-(3^2-3)+(4^2-4)?</cmath><math>\textbf{(A)} ~1 \qquad\textbf{(B)} ~2 \qquad\textbf{(C)} ~5 \qquad\textbf{(D)} ~8 \qqu
    15 KB (2,302 words) - 23:41, 14 April 2024
  • ...n a garden spade. In addition, each of the <math>900</math> residents owns a bag of candy hearts. There are <math>437</math> residents who own exactly t A list of positive integers has the following properties:
    8 KB (1,236 words) - 23:11, 12 March 2024
  • {{AMC10 Problems|year=2022|ab=A}} <math>\textbf{(A)}\ \frac{31}{10}\qquad\textbf{(B)}\ \frac{49}{15}\qquad\textbf{(C)}\ \frac{
    18 KB (2,662 words) - 02:08, 9 March 2024
  • <math>\mathrm{(A) \ } -10\qquad \mathrm{(B) \ } -3\qquad \mathrm{(C) \ } 5\qquad \mathrm{(D) ...th> plane in the shape of a circle, centered at <math>(5, 6)</math> with a a radius of <math>8</math>. The area that is in the field but above the line
    9 KB (1,450 words) - 18:33, 21 April 2020
  • ...ber and the negative of one of the solutions of <math>x^2-3x+c=0</math> is a solution of <math>x^2+3x-c=0</math>, then the solutions of <math>x^2-3x+c=0 <math>\textbf{(A) }1,~2\qquad \textbf{(B) }-1,~-2\qquad \textbf{(C) }0,~3\qquad \textbf{(D)
    807 bytes (143 words) - 20:11, 12 July 2020
  • Using casework on where the consecutive element pair is, there is a unique consecutive element pair that satisfies the conditions. It is easy t ...by <math>N_1 \left( m \right)</math> the number of subsets of a set <math>S</math> that consists of <math>m</math> consecutive integers, such that each
    15 KB (2,414 words) - 06:57, 26 November 2023
  • ...<math>r</math> lies within both cones. The maximum possible value of <math>r^2</math> is <math>\frac{m}{n}</math>, where <math>m</math> and <math>n</mat ...and thus must be at the intersection of their axes of symmetry. Let <math>A</math> be the point in the cross section where the bases of the cones meet,
    7 KB (951 words) - 10:11, 26 December 2023
  • ...hese three common roots are distinct. If <math>Q(0) = 2</math>, then <math>R(0) = \frac{m}{n}</math>, where <math>m</math> and <math>n</math> are relati Let <math>Q(x) = x^2 + ax + 2</math> and <math>R(x) = x^2 + bx + c</math>. We can write the following:
    6 KB (1,015 words) - 21:16, 28 December 2023
  • ...h>, and <math>EA=7</math>. Moreover, the pentagon has an inscribed circle (a circle tangent to each side of the pentagon). Find the area of <math>ABCDE< ...the area of triangle <math>ANM</math> is 108. Triangle <math>END</math> is a triangle with side lengths 6, 8, 10, so the area of two of them is 48, so t
    13 KB (2,197 words) - 23:00, 8 January 2024
  • SINCE MY COMPUTER WON'T LOAD THIS FOR SOME REASON, I'LL BE UPDATING THIS AS I GO THOUGH :) ...o this thing is almost the exact same format as Lcz's :P ). (Ok, actually, a LOT of credits to Lcz)
    4 KB (671 words) - 13:59, 22 July 2020
  • ...dius equal to <math>\frac{1}{2}AB</math>, meeting <math>AB</math> at <math>T</math>. Then <math>AT</math> and <math>TB</math> are the roots of: <math>\textbf{(A)}\ x^2+px+q^2=0\\ \textbf{(B)}\ x^2-px+q^2=0\\ \textbf{(C)}\ x^2+px-q^2=0\\
    971 bytes (170 words) - 16:43, 7 July 2020
  • ...pectively, intersect at distinct points <math>A</math> and <math>B</math>. A third circle <math>\omega</math> is externally tangent to both <math>\omega <font color=red><b>Claim.</b></font color> <math>DPEQ</math> is a harmonic quadrilateral.
    17 KB (2,852 words) - 03:59, 7 February 2024
  • ...1</math> is divisible by <math>2021</math> for all positive integers <math>a</math>. Find the sum of the prime factors in the prime factorization of <ma .... By Dirichlet's Theorem (Refer to the <b>Remark</b> section.), such <math>a</math> always exists.
    13 KB (2,185 words) - 02:28, 13 November 2023
  • ...m of the least element of <math>S</math> and the greatest element of <math>S</math>. ...with the vertex at <math>(0,-k),</math> and <math>x=2(y-20)^2-k</math> is a rightward-opening parabola with the vertex at <math>(-k,20).</math> We cons
    10 KB (1,742 words) - 02:31, 13 November 2023
  • {{AMC12 Problems|year=2021|ab=A}} <math>\textbf{(A) }0 \qquad \textbf{(B) }50 \qquad \textbf{(C) }52 \qquad \textbf{(D) }54 \q
    15 KB (2,383 words) - 09:49, 25 June 2023
  • ...th>ABCD</math> be a parallelogram with <math>\angle BAD < 90^\circ.</math> A circle tangent to sides <math>\overline{DA},</math> <math>\overline{AB},</m pair A,B,C,D,P,Q;
    16 KB (2,517 words) - 20:22, 31 January 2024
  • S_n = \sum | A \cap B | , ...ubsets of <math>\left\{ 1 , 2 , 3, \cdots , n \right\}</math> with <math>|A| = |B|</math>.
    9 KB (1,471 words) - 16:41, 1 February 2024
  • ...h> into two polygons of equal perimeter. Let <math>\triangle ABC</math> be a triangle where <math>BC = 219</math> and <math>AB</math> and <math>AC</math ...mple angle chasing reveals the condition is now equivalent to <math>\angle A=120^\circ</math>.
    16 KB (2,730 words) - 02:56, 4 January 2023
  • <math> \textbf{(A)}\ {-}2 \qquad real r = 3, s = 5, t = sqrt(r*r+s*s);
    15 KB (2,233 words) - 13:02, 10 November 2023
  • <math>(\textbf{A})\: 10{,}000\qquad(\textbf{B}) \: 10{,}010\qquad(\textbf{C}) \: 10{,}110\qq pen s = linewidth(0.8)+fontsize(8);
    16 KB (2,450 words) - 00:13, 12 November 2023
  • {{AMC10 Problems|year=2021 Fall|ab=A}} <math>\textbf{(A) } 7 \qquad\textbf{(B) } 21 \qquad\textbf{(C) } 49 \qquad\textbf{(D) } 64 \
    14 KB (2,162 words) - 21:33, 2 November 2023
  • ...rger square as shown. The sum of all possible values for the area of <math>R</math> can be written in the form <math>\tfrac mn</math>, where <math>m</ma label("$3$",(11/2,9/2),S);
    16 KB (2,274 words) - 09:02, 10 December 2022
  • A cube is constructed from <math>4</math> white unit cubes and <math>4</math> <math>\textbf{(A)}\ 7 \qquad\textbf{(B)}\ 8 \qquad\textbf{(C)}\ 9 \qquad\textbf{(D)}\
    10 KB (1,641 words) - 22:17, 26 September 2023
  • In a triangle <math>ABC</math> we have <math>AB = AC.</math> A circle which is internally tangent with the circumscribed circle of the tri ...{s - a}{b} \cdot \frac{2R}{h}</math> Denote <math>h' = AL</math> the <math>A</math>-altitude of the triangle <math>\triangle APQ</math>. Then
    3 KB (541 words) - 17:04, 29 January 2021
  • ...positive<math>.</math> The <math>n-th</math> positive integer which doesn't belong to the sequence is <math>f(f(n))+1.</math> Find <math>f(240).</math> ...h>-th number missing is <math>f(f(n))+1</math> and <math>f(f(n))</math> is a member of the sequence, it results that there are exactly <math>n-1</math>
    2 KB (448 words) - 17:01, 29 January 2021
  • ...ts are that the difference between the number of tenors and basses must be a multiple of <math>4</math>, and the group must have at least one singer. Le <math>\textbf{(A) } 47\qquad\textbf{(B) } 48\qquad\textbf{(C) } 83\qquad\textbf{(D) } 95\qqu
    7 KB (1,152 words) - 14:13, 29 February 2024
  • <math>\textbf{(A) }7 \qquad \textbf{(B) }9 \qquad \textbf{(C) }11 \qquad \textbf{(D) }13\qqu ...triangle formed by the points <math>P_1 = (r,0)</math>, <math>P_2 = \left(-r,\sqrt 3\right)</math>, and <math>P_3 = \left(-2r,\sqrt 2\right)</math> lies
    11 KB (1,858 words) - 18:33, 1 August 2022
  • <math>\textbf{(A) }{-}\frac{3}{49} \qquad \textbf{(B) }{-}\frac{1}{28} \qquad \textbf{(C) }\ ==Solution 1 (Complex Numbers: Vieta's Formulas)==
    9 KB (1,484 words) - 02:25, 21 September 2023
  • ...a positive integer not divisible by the square of any prime. What is <math>a+b+c</math>? <math>\textbf{(A) }110 \qquad \textbf{(B) }114 \qquad \textbf{(C) }118 \qquad \textbf{(D) }1
    16 KB (2,539 words) - 07:30, 30 December 2023
  • pair[] A; for (int i=0; i<7; ++i) { A[i] = rotate(60*i)*(1,0);}
    5 KB (865 words) - 07:14, 24 January 2024
  • Luka is making lemonade to sell at a school fundraiser. His recipe requires <math>4</math> times as much water a <math>\textbf{(A) } 6\qquad\textbf{(B) } 8\qquad\textbf{(C) } 12\qquad\textbf{(D) } 18\qquad
    20 KB (2,925 words) - 03:42, 24 January 2024
  • ...r <math>n \geqslant 3</math> is called frameable if it is possible to draw a regular polygon with <math>n</math> sides all whose vertices lie on these l (a) Show that <math>3, 4, 6</math> are frameable.
    3 KB (607 words) - 03:12, 22 September 2023
  • ==Generalized Wooga Looga Theorem (The Devil's Triangle)== ...C]}=1-\frac{r(s+1)+s(t+1)+t(r+1)}{(r+1)(s+1)(t+1)}=\frac{rst+1}{(r+1)(s+1)(t+1)}</math>.
    3 KB (582 words) - 18:05, 23 March 2024
  • ...p paths are there from <math>P</math> to <math>Q?</math> (The figure shows a sample path.) <math>\textbf{(A) }28 \qquad \textbf{(B) }30 \qquad \textbf{(C) }32 \qquad \textbf{(D) }33 \
    7 KB (1,170 words) - 22:50, 19 January 2024
  • A '''quartic equation''' is an algebraic [[equation]] of the form ...ver, there are very clever methods for solving them by bringing it down to a [[Cubic Equation|cubic]]. I am going to list the simplest of the five. Also
    13 KB (2,376 words) - 16:56, 19 February 2024
  • ...ommon tangent that passes through <math>P</math>. If this tangent is also a common tangent to <math>C_2</math> and <math>C_1</math>, find the radius of ...math>b</math>, <math>b+1</math> evenly divides <math>c</math>, and <math>c-a=10</math>.
    7 KB (1,149 words) - 17:16, 15 December 2020
  • In a narrow alley of width <math>w</math> a ladder of length <math>a</math> is placed with its foot at point <math>P</math> between the walls. ...wall at <math>Q</math>, the distance <math>k</math> above the ground makes a <math>45^\circ</math> angle with the ground.
    2 KB (383 words) - 22:47, 13 September 2021
  • The '''functional equation for Riemann zeta function''' is a result due to analytic continuation of [[Riemann zeta function]]: \zeta(s)=2^s\pi^{s-1}\sin\left(\pi s\over2\right)\Gamma(1-s)\zeta(1-s)
    4 KB (682 words) - 03:56, 13 January 2021
  • <math>\textbf{(A) }-1\qquad We can rewrite the equation as <math>x^2 + 2hx - 3 = 0.</math> By Vieta's Formulas, the sum of the roots is <math>-2h</math> and the product of the r
    828 bytes (143 words) - 20:09, 28 January 2021
  • ...and <math>\overline{CD}</math> with <math>AB>CD.</math> Line segments from a point inside <math>ABCD</math> to the vertices divide the trapezoid into fo pair A=(-1, 0), B=(1, 0), C=(0.3, 0.9), D=(-0.3, 0.9), P=(0.2, 0.5), E=(0.1, 0.75)
    3 KB (484 words) - 12:58, 10 September 2023
  • ...nd the degree of <math>R</math> is less than <math>2.</math> What is <math>R(z)?</math> <math>\textbf{(A) }{-}z \qquad \textbf{(B) }{-}1 \qquad \textbf{(C) }2021\qquad \textbf{(D)
    8 KB (1,315 words) - 11:43, 24 October 2023
  • ...h> where <math>1<a<b<c.</math> What is <math>g(1)</math> in terms of <math>a,b,</math> and <math>c?</math> ...2}\qquad \textbf{(D) }\frac{a+b+c}{c^2} \qquad \textbf{(E) }\frac{1+a+b+c}{a+b+c}</math>
    4 KB (720 words) - 10:02, 1 October 2023
  • Each of the vertices of a convex hexagon is a center of a circle of radius equal to the not longer side of the hexagon which includes ...elements <math>A,B</math> of that subset there exists a point <math>C \in S</math> such that the area of triangle <math>ABC</math> is equal to <math>k<
    2 KB (377 words) - 18:08, 4 July 2022
  • ...gcd(r,s)=1,}</math> then <math>\boldsymbol{\gcd(r,t)\cdot\gcd(s,t)=\gcd(rs,t).}</math></b> As <math>r</math> and <math>s</math> are relatively prime (have no prime divisors in common), this proper
    7 KB (1,212 words) - 15:54, 15 April 2024
  • triple O1, O2, O3, T1, T2, T3, A, L1, L2; A = (0,0,-36*sqrt(1105)/23);
    13 KB (2,124 words) - 01:00, 14 January 2023
  • ...st positive integer <math>n</math> for which <math>2^n + 5^n - n</math> is a multiple of <math>1000</math>. ...on to be true, <math>2^n \equiv n \pmod{5}</math> is necessary; it shouldn't take too long for us to go through the <math>20</math> possible LHS-RHS com
    16 KB (2,240 words) - 23:16, 26 January 2024
  • <math>\textbf{(A)}\ 42\qquad\textbf{(B)}\ 43\qquad\textbf{(C)}\ 44\qquad\textbf{(D)}\ 45\qqu <math>\textbf{(A)}\ 0\qquad\textbf{(B)}\ 1\qquad\textbf{(C)}\ 2\qquad\textbf{(D)}\ 3\qquad\t
    4 KB (675 words) - 14:01, 28 May 2021
  • <math>\textbf{(A)}\ -2\qquad\textbf{(B)}\ -1\qquad\textbf{(C)}\ 0\qquad\textbf{(D)}\ 1\qquad ...h> with equal chances. What is the probability that the number selected is a perfect square, given <math>n=4</math>?
    8 KB (1,385 words) - 12:55, 23 June 2021
  • When <math>30\%</math> of <math>x</math> is a positive perfect square integer, what is <math>min(x)</math> such that <mat <math>\textbf{(A)} ~9 \qquad\textbf{(B)} ~12 \qquad\textbf{(C)} ~30 \qquad\textbf{(D)} ~36 \
    11 KB (1,691 words) - 18:56, 25 April 2022
  • <math>\textbf{(A)}\ 0 \qquad\textbf{(B)}\ 1 \qquad\textbf{(C)}\ 2 \qquad\textbf{(D)}\ ...> have magnitude of <math>1</math>, so the answer is <math>\boxed{\textbf{(A)}\ 0 }</math> ~lopkiloinm
    6 KB (1,078 words) - 17:50, 19 May 2024
  • ...th side length <math>6.</math> The five vertices of the pyramid all lie on a sphere with radius <math>\frac mn</math>, where <math>m</math> and <math>n< triple A = (-3,-3,0);
    4 KB (649 words) - 00:33, 29 January 2024
  • Let <math>ABCD</math> be a convex quadrilateral with <math>AB=2, AD=7,</math> and <math>CD=3</math> su pair A,B,C,D,M,H; real xb=71, xd=121;
    14 KB (2,254 words) - 18:26, 8 February 2024
  • ...h>PB</math> is tangent to semicircle <math>RBT</math>; <math>SRT</math> is a straight line; pair O1=(0,0), O2=(3,0), Sp=(-2,0), R=(2,0), T=(4,0);
    2 KB (336 words) - 02:02, 14 October 2021
  • <math> \textbf{(A)}\ {-}2 \qquad real r = 3, s = 5, t = sqrt(r*r+s*s);
    15 KB (2,224 words) - 13:10, 20 February 2024
  • ...tegers in <math>\{-20,-19,-18,\ldots,18,19,20\},</math> such that there is a unique integer <math>m \not= 2</math> with <math>p(m) = p(2).</math> ...what we just need to do is to just find the number of ordered pairs <math>(a, b)</math> that work, and multiply it by <math>41.</math>
    5 KB (960 words) - 20:40, 27 December 2023
  • ...real numbers. Among all the disrespectful quadratic polynomials, there is a unique such polynomial <math>\tilde{p}(x)</math> for which the sum of the r <math>\textbf{(A) } \dfrac{5}{16} \qquad\textbf{(B) } \dfrac{1}{2} \qquad\textbf{(C) } \dfra
    26 KB (4,697 words) - 22:19, 2 January 2024
  • A regular hexagon of side length <math>1</math> is inscribed in a circle. Each minor arc of the circle determined by a side of the hexagon is reflected over that side. What is the area of the re
    6 KB (1,021 words) - 19:40, 1 November 2023
  • ...}</math> and <math>\overline{CQ}</math> intersect at right angles at <math>R</math>, with <math>BR = 6</math> and <math>PR = 7</math>. What is the area real r = 3.5;
    11 KB (1,733 words) - 23:06, 13 May 2024
  • <math>\textbf{(A)}\: 10{,}000\qquad\textbf{(B)} \: 10{,}010\qquad\textbf{(C)} \: 10{,}110\qq pen s = linewidth(0.8)+fontsize(8);
    14 KB (2,191 words) - 19:57, 12 November 2023
  • ...be a convex pentagon such that <math>BC = DE</math>. Assume that there is a point <math>T</math> inside <math>ABCDE</math> with <math>TB = TD</math>, <math>TC = TE</
    2 KB (307 words) - 01:55, 19 November 2023
  • ...amber cells and <math>b</math> bronze cells such that no two of the <math>a+b</math> chosen cells lie in the same row or column. ...>2b</math>-gon <math>B</math> is formed by translating the black rods, and a convex <math>2w</math>-gon <math>W</math> is formed by translating the whit
    4 KB (762 words) - 13:45, 22 November 2023
  • ...guess can cause the recursion to cycle or diverge instead of converging to a root. ...exactly one root, as long as <math>f'(x_i) \neq 0</math> we can construct a [[Taylor polynomial#Tangent-line approximation|tangent-line approximation]]
    13 KB (2,298 words) - 23:34, 28 May 2023
  • ...eq t</math> are real numbers such that <math>f(s)\neq 0</math> and <math>f(t)\neq 0</math>. {{USAMO newbox|year=2022|num-b=4|num-a=6}}
    671 bytes (113 words) - 15:00, 27 March 2022
  • ...n of the linear transformation's image, and nullity is the dimension of it's kernel (the word nullity comes from the term "null space" which is used to <center><math>\text{nullity}(\text{T})+\text{rank}(\text{T})=\dim(\text{T}).</math></center>
    5 KB (893 words) - 22:41, 28 May 2022
  • A focused asymptote reference with examples and short code.<p style="margin-l <i>Well, it isn't the perfect graphics package, but we do think it is getting there asympto
    28 KB (4,808 words) - 19:43, 3 June 2022
  • <math>\textbf{(A)}\ 0 \qquad \textbf{(B)}\ 6 \qquad \textbf{(C)}\ 10 \qquad \textbf{(D)}\ 18 A square piece of paper is folded twice into four equal quarters, as shown be
    29 KB (4,048 words) - 17:25, 21 February 2024
  • ...s <math>\frac{1}{4}</math> of the lattice points contained in <math>R \cup S.</math> See the figure (not drawn to scale). label(scale(1.3)*"$R$", (55/2,55/2));
    8 KB (1,433 words) - 12:02, 30 April 2023
  • Let <math>P(x)</math> be a polynomial with rational coefficients such that when <math>P(x)</math> is d is <math>2x+1</math>. There is a unique polynomial of least degree with these two properties. What is the su
    7 KB (1,219 words) - 23:07, 19 September 2023
  • real r = 3, s = 5, t = sqrt(r*r+s*s); pair A = (0,0), B = (r,s), C = (r+t,s), D = (t,0), P = (r,0);
    3 KB (498 words) - 17:14, 29 June 2023
  • <math> \textbf{(A)}\ -2 \qquad ...e base of the exponents are the roots of the characteristic equation <math>r^3-1=0</math>. So we have
    5 KB (866 words) - 22:17, 27 October 2023
  • ...t lines to the reference triangle's circumcircle at the reference triangle's vertices). ...le ABC.</math> Let <math>\Omega'</math> be circumcircle of <math>\triangle A'B'C'.</math>
    2 KB (297 words) - 14:14, 18 November 2022
  • ...th> and <math>\triangle ABC</math> are concurrent at Schiffler point <math>S = X(21), \frac {OS}{SG} = \frac {3R}{2r}</math>. ...ler line <math>OG</math> of <math>\triangle ABC</math> at such point <math>S,</math> that <math>\frac {OS}{SG} = \frac {3R}{2r}</math>.
    3 KB (576 words) - 16:41, 23 November 2022
  • ...oday, after walking <math>5</math> blocks, Viswam discovers he has to make a detour, walking <math>3</math> blocks of equal length instead of <math>1</m real d,s; // gap and side
    6 KB (1,028 words) - 17:51, 11 May 2024
  • <math>\textbf{(A) } 0\qquad\textbf{(B) } 2\qquad\textbf{(C) } 4\qquad\textbf{(D) } 6\qquad\t <math>\textbf{(A) } 6.4\qquad\textbf{(B) } 6.504\qquad\textbf{(C) } 6.54\qquad\textbf{(D) }
    21 KB (2,856 words) - 17:51, 8 May 2024
  • ...y <math>\text{P}</math>s, <math>\text{Q}</math>s, and <math>\text{R}</math>s will appear in the completed table? label(scale(.9)*"\textsf{R}", (.5,2.5));
    13 KB (1,813 words) - 03:45, 24 January 2024
  • Rhombus <math>ABCD</math> has <math>\angle BAD < 90^\circ.</math> There is a point <math>P</math> on the incircle of the rhombus such that the distances pair A, B, C, D, O, P, R, S, T;
    17 KB (2,612 words) - 14:54, 3 July 2023
  • Because the answer must be a positive integer, it is just equal to the modulus of the product. Define <m ...roduct (two at a time, where <math>1</math> is treated as the identity) as a series of arrays:
    9 KB (1,284 words) - 23:37, 31 January 2024
  • ...result in number theory that implies many other theorems, such as [[Euclid's Lemma]] and the [[Chinese Remainder Theorem]]. ...ers <math>x</math> and <math>y</math> such that <math>15x + 6y = 3</math>. A solution <math>(x, y)</math> of this equation is <math>(1, -2)</math>.
    3 KB (562 words) - 11:19, 5 March 2023
  • ...ath>\ell_a</math> passes through the feet of the perpendiculars from <math>A</math> to <math>DB</math> and <math>DC</math>. Line <math>m_a</math> passes ...<math>b = a_k</math>. Find the maximum possible number of forced terms in a regular sequence with <math>1000</math> terms.
    5 KB (873 words) - 08:45, 24 February 2023
  • ...epeatedly sliding dominoes. Find the maximum value of <math>k(C)</math> as a function of <math>n</math>. ...<math>B</math> decides to end the game. At this point, <math>B</math> gets a score, given by the number of unit squares in the largest (in terms of area
    4 KB (681 words) - 21:38, 19 March 2024
  • ...\rightarrow\mathbb{R}^{+}</math> such that, for all <math>x, y \in \mathbb{R}^{+}</math>, ...eatedly sliding dominoes. Find all possible values of <math>k(C)</math> as a function of <math>n</math>.
    4 KB (664 words) - 22:57, 9 March 2024
  • ...epeatedly sliding dominoes. Find the maximum value of <math>k(C)</math> as a function of <math>n</math>. ...is even but its column is odd, a tile B if its row and column is even, and a tile Y if its row is odd but its column is even.
    5 KB (867 words) - 19:18, 6 October 2023
  • ...distinct element in the range) <math>f:S\rightarrow S,</math> where <math>S=\{1, 2, 3, 4, 5\}</math>. What is the average of <math>f(1) + f(2) + f(3)</ ...and a point <math>P</math> in the same plane as the base of the cone, but a distance of <math>10</math> from the center of the base of the cone. We rot
    5 KB (830 words) - 13:04, 14 December 2023
  • ...r. A chain is any subsequence of consecutive coins of the same type. Given a fixed positive integer <math>k\le 2n</math>, Marianne repeatedly performs t ...<math>x \in \mathbb{R}^+</math>, there is exactly one <math>y \in \mathbb{R}^+</math> satisfying
    3 KB (605 words) - 15:19, 3 June 2023
  • ...ometers per hour, how many minutes will the walk take, including the <math>t</math> minutes at the coffee shop? ...t there is a strategy that guarantees that Bob wins, regardless of Alice’s moves.
    8 KB (1,307 words) - 20:00, 6 February 2024
  • ...ath> at <math>D</math> and meets <math>\Omega</math> again at <math>E \neq A</math>. The line through <math>D</math> parallel to <math>BC</math> meets l ...e the point diametrically opposite to a point <math>S</math> through <math>S' \implies AS'</math> is the internal angle bisector of <math>\angle BAC</ma
    7 KB (1,318 words) - 06:38, 1 May 2024
  • ...with the following property: there exists two points <math>Q</math>, <math>R</math> on <math>l</math> such that <math>M</math> is the midpoint of <math> ...have been shortened in this solution. The solution in the video provides a much faster solution,
    6 KB (1,052 words) - 00:42, 17 November 2023
  • ...ath>B'(4, 3)</math> about the point <math>P(r, s)</math>. What is <math>|r-s|</math>? <math>\textbf{(A) } \frac{1}{4} \qquad \textbf{(B) } \frac{1}{2} \qquad \textbf{(C) } \frac{
    5 KB (817 words) - 11:51, 19 May 2024
  • Each square in a <math>3\times3</math> grid of squares is colored red, white, blue, or green label("R", (19.5,1.5));
    8 KB (1,261 words) - 15:23, 6 February 2024
  • In a table tennis tournament every participant played every other participant ex <math>\textbf{(A) }15\qquad\textbf{(B) }36\qquad\textbf{(C) }45\qquad\textbf{(D) }48\qquad\t
    5 KB (810 words) - 22:56, 10 May 2024
  • ...th>\theta = \angle PAB=\angle QPC=\angle RQB=\cdots</math> will produce in a length that is <math>120</math> meters? (This figure is not drawn to scale. label("$A$",(-58,-15));
    7 KB (1,074 words) - 20:58, 19 May 2024
  • pair A = (0,0), B = (-0.25,0), C = (0.25,0), D = (0,6/7), E = (-0.95710678118, 0.7 <math>\textbf{(A) } \frac{1}{14} \qquad \textbf{(B) } \frac{1}{12} \qquad \textbf{(C) } \fra
    4 KB (527 words) - 23:54, 20 April 2024
  • ...tices of the pentagon are folded into the center of the pentagon, creating a smaller pentagon. What is the area of the new pentagon? <math>\textbf{(A)}~4-\sqrt{5}\qquad\textbf{(B)}~\sqrt{5}-1\qquad\textbf{(C)}~8-3\sqrt{5}\qqu
    19 KB (2,967 words) - 16:56, 24 February 2024
  • ==Polya’s method== Natural scientists have found a number of empirical laws and methods that describe nature and therefore opt
    12 KB (2,104 words) - 14:11, 24 February 2024
  • ...How many of these triangles are equilateral and contain <math>P</math> as a vertex? pair P,Q,R,S,T,U,V,W;
    3 KB (426 words) - 01:50, 18 April 2024
  • <math>\textbf{(A)}~11\qquad\textbf{(B)}~6\qquad\textbf{(C)}~8\qquad\textbf{(D)}~10\qquad\tex We start by trying to prove a function of <math>n</math>, and then we can apply the function and equate i
    13 KB (2,194 words) - 19:10, 18 December 2023
  • ...part of <cmath>(75+117i)z+\frac{96+144i}{z}</cmath>where <math>z</math> is a complex number with <math>|z|=4</math>. ==Video Solution: Cauchy's Inequality==
    12 KB (1,957 words) - 09:04, 18 May 2024
  • ...ar boxes that are elements of <math>\mathcal{B}</math>. The value of <math>r^2</math> can be written as <math>\frac{p}{q}</math>, where <math>p</math> a ...make the length longer.) Thus, let the width and height be of length <math>a</math> and the length be <math>L</math>.
    10 KB (1,554 words) - 22:26, 13 April 2024
  • If <math>r</math>, <math>s</math>, and <math>t</math> are all the roots of the equation: (a) Prove that <math>r</math>, <math>s</math>, and <math>t</math> are all postive
    581 bytes (98 words) - 13:27, 13 December 2023
  • ...th>G</math>, the sum of the squares of its distances to the vertices <math>A</math>, <math>B</math> and <math>C</math> is 5. ...r every point <math>P</math> in <math>G</math> it is possible to construct a triangle whose sides have the lengths of the segments <math>AP</math>, <mat
    4 KB (814 words) - 09:42, 23 December 2023
  • ...be their common perpendicular, where <math>A \in r</math>, and <math>B \in s</math> '''(*)'''. ...e condition that <math>MN</math> is tangent to the sphere at a point <math>T</math>.
    767 bytes (136 words) - 14:48, 13 December 2023
  • ...ath>F</math> and <math>R</math> (<math>F</math> being the closest to <math>A</math>). ...section of the diagonals of the quadrilateral <math>EQFR</math>. Let <math>T</math> be the point of intersection of the diagonals of the <math>FRDP</mat
    1 KB (202 words) - 15:23, 13 December 2023
  • ...th>r,s,t.</math> A monic polynomial <math>g</math> has roots <math>r^3,s^3,t^3.</math> Given that the minimum possible value of <math>\frac{g(1)}{f(1)}<
    362 bytes (64 words) - 13:03, 14 December 2023
  • ...2, 3)</math>. Find the largest possible value of <math>a^b + b^c + c^d + d^a</math> ...nd increases in altitude at an angle of <math>30^{\circ}.</math> Let <math>s</math> the speed in feet per second that the plane must fly at to move nort
    8 KB (1,238 words) - 02:14, 3 January 2024
  • ...<math>D,E,C,F</math> are collinear. Also, <math>A,D,H,G</math> all lie on a circle. If <math>BC=16</math>,<math>AB=107</math>,<math>FG=17</math>, and < pair A = (0,0);pair B = (70,0);pair C = (70,16);pair D = (0,16);pair E = (3,16);pa
    7 KB (1,144 words) - 17:43, 22 April 2024
  • ...m <math>E</math> to the edges <math>AB, BC, CA.</math> Then <math>\frac{s}{S}</math> equals \text{(A) }\sqrt{2}
    2 KB (308 words) - 19:04, 10 March 2024
  • ...h>12</math>-gon) where each side of the rectangle lies on either a side or a diagonal of the dodecagon. The diagram below shows three of those rectangle ...l (don't forget to multiply by 3 at the end of Case 1). Then the rectangle's sides coincide with these segments as shown in the diagram.
    8 KB (1,395 words) - 17:26, 9 February 2024
  • ...State University named after M.V. Lomonosov. The first four problems have a standard level. ...le of its upper base touches each of the faces which contains vertex <math>S.</math>
    32 KB (5,375 words) - 11:43, 5 May 2024
  • This is a page where you can share the problems you made (try not to use past exams). <cmath>\frac{ab+bc+ac}{(a+b+c)^2}</cmath>
    43 KB (7,006 words) - 14:24, 19 February 2024